usmle questionnarie

104
A 17-year-old female presents to the university health clinic stating she has yet to experience menses despite developing breasts 3 years ago. Her previous medical history is unremarkable, and she has never been sexually active. On examination, her height is at the 65th percentile, weight is at the 50th percentile, and blood pressure is 110/70 mm Hg. Breasts exhibit Tanner stage IV development. Vaginal examination demonstrates a short, blind vaginal pouch and pelvic ultrasound fails to locate a uterus. What is the next best step in the diagnosis of this patient? A . Brain magnetic resonance imaging (MRI) B . Serum beta human chorionic gonadotropin (β-HCG) C . Serum follicle-stimulating hormone (FSH) and karyotype D . Serum progesterone E . Serum testosterone and karyotype QID: 33090 Option E (Serum testosterone and karyotype) is correct. In a patient presenting with primary amenorrhea (no menses by age 16 with some secondary sexual characteristics present) and an absent uterus on ultrasound, the most appropriate first tests are serum testosterone and karyotype analysis. The differential diagnosis in breast present, uterus absent primary amenorrhea is Müllerian agenesis and androgen insensitivity. This patient is exhibiting characteristics consistent with both. Female-appropriate testosterone levels will suggest a diagnosis of Müllerian agenesis and a karyotype of 46 XX will confirm the diagnosis. If this patient had male-appropriate testosterone, we would consider a diagnosis of androgen insensitivity and a karyotype of 46 XY would confirm the diagnosis.

Transcript of usmle questionnarie

Page 1: usmle questionnarie

A 17-year-old female presents to the university health clinic stating she has yet to experience

menses despite developing breasts 3 years ago. Her previous medical history is unremarkable,

and she has never been sexually active. On examination, her height is at the 65th percentile,

weight is at the 50th percentile, and blood pressure is 110/70 mm Hg. Breasts exhibit Tanner

stage IV development. Vaginal examination demonstrates a short, blind vaginal pouch and pelvic

ultrasound fails to locate a uterus. What is the next best step in the diagnosis of this patient?

A. Brain magnetic resonance imaging (MRI)

B. Serum beta human chorionic gonadotropin (β-HCG)

C. Serum follicle-stimulating hormone (FSH) and karyotype

D. Serum progesterone

E. Serum testosterone and karyotype

QID: 33090

Option E (Serum testosterone and karyotype) is correct. In a patient presenting with primary amenorrhea (no menses by age 16 with some secondary sexual characteristics present) and an absent uterus on ultrasound, the most appropriate first tests are serum testosterone and karyotype analysis. The differential diagnosis in breast present, uterus absent primary amenorrhea is Müllerian agenesis and androgen insensitivity. This patient is exhibiting characteristics consistent with both. Female-appropriate testosterone levels will suggest a diagnosis of Müllerian agenesis and a karyotype of 46 XX will confirm the diagnosis. If this patient had male-appropriate testosterone, we would consider a diagnosis of androgen insensitivity and a karyotype of 46 XY would confirm the diagnosis.

Option A (Brain magnetic resonance imaging [MRI]) is incorrect. Brain magnetic resonance imaging (MRI) would be appropriate in a patient in whom we suspected Kallmann syndrome or a tumor. In this patient without a uterus, endocrinologic evaluation should be performed prior to an MRI.

Option B (Serum beta human chorionic gonadotropin [β-HCG]) is incorrect. In a patient who has never been sexually active, has a blind vagina on examination, and does not have a uterus as demonstrated by ultrasound, a pregnancy test would certainly be negative.

Option C (Serum follicle-stimulating hormone [FSH] and karyotype) is incorrect. Serum follicle-stimulating hormone (FSH) and karyotype are the most appropriate next investigations in a patient with primary amenorrhea when a uterus is present and pregnancy via beta human chorionic gonadotropin (β-hCG) measurement has been ruled out. Elevated FSH suggests primary ovarian failure and makes our differential diagnosis between Turner syndrome, vanishing testes syndrome, or absence of testes determining factor. Turner syndrome is much more common than the other two and can be distinguished via karyotyping. Low FSH suggests a hypothalamic-pituitary problem, such as Kallmann syndrome (low gonadotropin-releasing hormone [GnRH] and anosmia).

Page 2: usmle questionnarie

Option D (Serum progesterone) is incorrect. The rare 17-alpha hydroxylase deficiency form of congenital adrenal hyperplasia results in androgen and estrogen deficiency with an increase in serum progesterone and blood pressure. This patient has normal blood pressure.

PRIMARY AMENORRHEA WITH BREAST DEVELOPMENT AND MÜLLERIAN ANOMALIES

Patients with primary amenorrhea, breast development, and Müllerian anomalies all fail to demonstrate a visible or palpable uterine cervix on physical examination. They fall into two categories: those with complete androgen insensitivity syndrome (46 XY) and those with a karyotype of 46 XX. The distinction can be made by the serum testosterone level. Patients with complete androgen insensitivity syndrome have male levels of testosterone.

The karyotype in patients with androgen insensitivity syndrome is 46 XY, and they have testes that are often intraabdominal. Breast development (with smaller nipples and areolae than normal) is caused by an enzymatic conversion of male levels of androgen to estrogen. The testes in these patients secrete normal male amounts of Müllerian-inhibiting substance; hence, patients have only a vaginal dimple and no uterus or tubes. Treatment should consist of gonadal resection to avoid malignant neoplasia once puberty is complete and the creation of a neovagina when the patient is prepared to be sexually active. Psychological counseling is also an important component in the care of these patients.

From Essentials of Obstetrics & Gynecology 4E by Hacker et al

Patients with primary amenorrhea, breast development, and a karyotype of 46 XX with anatomical anomalies have levels of testosterone appropriate for females. One should suspect an imperforate hymen in adolescents who report monthly dysmenorrhea in the absence of menstrual flow. On examination, these patients often present with a vaginal bulge and a midline cystic mass on rectal examination. Ultrasonography confirms the presence of a normal uterus and ovaries with hematocolpos. These patients can be successfully treated by hymenectomy.

Alternatively, women may present with similar symptoms but no lower vaginal bulge. When ultrasonography confirms a normal uterus and ovaries, one should suspect the possibility of a transverse, obstructing vaginal septum or cervical agenesis. MRI is the diagnostic method of choice in these patients. If the MRI scan confirms a transverse septum, surgical correction is indicated. These procedures can be extremely difficult, and the surgeon must be prepared to use tissue expanders, split-thickness skin grafts, or other techniques to effect a functional vagina. Surgical construction of a functional cervix is extremely unlikely. In general, it is recommended that these women undergo hysterectomy.

From Essentials of Obstetrics & Gynecology 4E by Hacker et al

Finally, rectal examination and ultrasonography may show the absence of a uterus indicating Meyer-Rokitansky-Küster-Hauser syndrome. This syndrome is characterized by a failure of the Müllerian ducts to fuse distally and to form the upper genital tract. These patients usually have bilateral rudimentary uterine tissues (anlagen), fallopian tubes, and ovaries. It is uncommon to have functional endometrial tissue within the anlagen. On occasion, the ovaries are not visible on ultrasonography because they have not descended

Page 3: usmle questionnarie

into the pelvis. In these cases, computed tomography (CT) or MRI may identify them well above the pelvic brim.

Creation of a neovagina can be accomplished by using one of two general approaches. The Frank method of vaginal dilatation uses dilatation of the vaginal pouch with vaginal forms (usually thermoplastic acrylic resin [Lucite] dilators) over the course of weeks to months. Alternatively, a McIndoe vaginoplasty, which involves the surgical creation of a neovaginal space using a split-thickness skin graft, may be performed. Both of these methods should be initiated in proximity to the time when the patient anticipates having vaginal intercourse.

From Essentials of Obstetrics & Gynecology 4E by Hacker et al

A 29-year-old G2P1001 at 28 weeks’ gestation presents for a prenatal office visit. Her pregnancy

has been complicated by red cell sensitization with an anti-D titer of 1:128. Ultrasound reveals an

appropriately grown fetus with evidence of fetal hydrops, including scalp edema and ascites.

Which of the following is the most appropriate next step in patient care?

A. Amniocentesis

B. Cesarean delivery

C. Fetal blood sampling

D. Induction of labor

E. Repeat anti-D titer

QID: 26716

Option C (Fetal blood sampling) is correct. This fetus is demonstrating sonographic signs of hydrops due to anemia resulting from red cell sensitization. At this gestational age, the plan of management would be fetal blood sampling to confirm anemia and intrauterine blood transfusion.

Option A (Amniocentesis) is incorrect. This fetus already demonstrates sonographic signs of hydrops; an amniocentesis would not add any valuable information.

Option B (Cesarean delivery) is incorrect. Due to the early gestational age, the fetus should be treated in utero, rather than delivered.

Option D (Induction of labor) is incorrect. Due to the early gestational age, the fetus should be treated in utero, rather than delivered.

Option E (Repeat anti-D titer) is incorrect. This fetus already demonstrates sonographic signs of hydrops; a repeat anti-D titer would not add any valuable information.

Page 4: usmle questionnarie

ULTRASONIC DETECTION OF RH SENSITIZATION

Serial ultrasonic examinations of a woman with a fetus at risk for hemolytic disease can be a useful adjunct to amniocentesis in confirming fetal well-being and determining the advent of fetal hydrops. The examination should include a routine fetal assessment plus a determination of placental size and thickness and hepatic size. Both the placenta and the fetal liver are enlarged with hydrops. Fetal hydrops is easily diagnosed by the characteristic appearance of one or more of the following: ascites, pleural effusion, pericardial effusion, or skin edema. Appearance of any of these factors during an ultrasonic examination eliminates the need for diagnostic amniocentesis and necessitates therapeutic intervention based on fetal gestational age.

Doppler assessment of peak velocity in the fetal middle cerebral artery (MCA) may prove to be the most valuable ultrasonic tool for detecting fetal anemia. A value above 1.5 multiples of the median for gestational age is considered predictive. For accurate evaluation, the Doppler gate is placed over the fetal MCA just as it bifurcates from the carotid siphon. Color Doppler is clearly advantageous for this examination. After 35 weeks' gestation, this test may produce a higher false-positive rate (Figure 16-3).

PERCUTANEOUS UMBILICAL BLOOD SAMPLING

Figure 16-1 Modified Liley chart used to determine the appropriate management of the patient with isoimmunization. The ΔOD 450 nm level in the amniotic fluid at a given weeks' gestation determines whether fetal transfusion or delivery is advisable.

Figure 16-2 Queenan curve for ΔOD 450 values for the management of the patient with isoimmunization. OD, optical density; Rh, rhesus. Adapted from Queenan JT, Tomai TP, Ural SH, et al: Deviations in amniotic fluid optical density at a wavelength of

450 nm in Rh-immunized pregnancies from 14 to 40 weeks' gestation: A proposal for clinical management. Am J Obstet Gynecol 168:1370-1376, 1993.

Figure 16-3 Middle cerebral artery (MCA) Doppler peak velocities based on gestational age. MoM, multiples of the median. Data from Moise KJ, Jr: Management of Rhesus alloisoimmunization. Obstet Gynecol 100(3):600-611, 2002.

Table 16-1. Hematologic values for normal fetuses*

  GESTATIONAL AGE (WK)Hematologic Value 15 16-17 18-20 21-22 23-25 26-30Hgb, g/dL 10.9 ± 0.7 12.5 ± 0.8 11.48 ± 0.78 12.29 ± 0.89 12.4 ± 0.77 13.36 ± 1.18

RBCs, × 109/L 2.43 ± 0.26 2.68 ± 0.21 2.66 ± 0.29 2.97 ± 0.27 3.06 ± 0.27 3.52 ± 0.32

Page 5: usmle questionnarie

MCV, fL (±1) 143 ± 8 143 ± 12 133.9 ± 8.83 1.130 ± 6.17 126.2 ± 6.23 118.2 ± 5.7

*Values are for normal fetuses from 15 to 30 weeks' estimated gestational age.Data from American College of Obstetricians and Gynecologists: Management of isoimmunization in pregnancy. Technical Bulletin No. 148. Washington, DC, ACOG, 1990.Hgb, hemoglobin; RBCs, red blood cells; MCV, mean corpuscular volume.

Advances in fetal interventional techniques and high-resolution ultrasonography have made direct fetal blood sampling the most accurate method for the diagnosis of fetal hemolytic disease. Percutaneous umbilical blood sampling (PUBS) can allow measurement of fetal hemoglobin, hematocrit, blood gases, pH, and bilirubin levels. The hematologic values for normal fetuses from 15 to 30 weeks' gestation are listed in Table 16-1. The technique for fetal blood sampling is similar to that described for fetal intravenous transfusion discussed later in this chapter. One drawback to this diagnostic procedure is that it requires expertise above and beyond that required for amniocentesis. The major risk is fetal exsanguination from tears in placental vessels, but when performed by an experienced practitioner, the risk of this complication is only 2% or less. However, there is a greater risk of fetomaternal hemorrhage, reported to be as high as 40%. Percutaneous umbilical blood sampling should not be a first-line method of assessing fetal status unless clearly indicated.

A previously healthy, 18-month-old boy is brought to the emergency room with a cough and

severe shortness of breath with hypoxemia. He is admitted to the PICU, where he is diagnosed

with necrotizing pneumonitis. He had no known exposure to fumes or smoke. His 7-year-old

brother had recently suffered from a milder respiratory tract infection and conjunctivitis. Which of

the following conditions is the patient at higher risk for in the future?

A. Adenocarcinoma of the lung

B. Blindness from vasoproliferative retinal disease

C. Chronic bronchitis from enlargement of the airspaces distal to the terminal bronchioles

D. Immotile sperm and infertility

E. Obstruction of bronchioles and smaller bronchi by fibrotic masses of tissue

QID: 27605

Option E (Obstruction of bronchioles and smaller bronchi by fibrotic masses of tissue) is correct. This is descriptive of the later stages of bronchiolitis fibrosa obliterans, which follows cases of adenovirus pneumonitis in 30% to 60% of cases, depending on the population surveyed. The condition follows the destruction of the bronchiolar epithelium, which fills with cellular debris, followed by granulation tissue, which later becomes fibrotic. The condition may follow infection with adenovirus and other causes of viral pneumonitis as well as inhalation of fumes or foreign bodies or aspiration of amniotic fluid, lipids, or stomach acid.

Option A (Adenocarcinoma of the lung) is incorrect. There is no association between viral pneumonitis and lung cancer. Inhalation of toxic fumes may be associated with a higher

Page 6: usmle questionnarie

future risk of some kinds of cancer.

Option B (Blindness from vasoproliferative retinal disease) is incorrect. This describes the condition known as retinopathy of prematurity, which is a type of retinal blindness that afflicts premature infants and is associated with artificial oxygenation by a ventilator.

Option C (Chronic bronchitis from enlargement of the airspaces distal to the terminal bronchioles) is incorrect. This is a description of emphysema, such as that that would occur in a child with alpha1-antitrypsin deficiency.

Option D (Immotile sperm and infertility) is incorrect. Respiratory disorders that are associated with decreased fertility include cystic fibrosis (CF), due to failure of the normal formation of mesonephric duct structures, and Kartagener’s syndrome, due to immotile sperm.

240 Adenoviruses Kenneth McIntosh

Adenoviruses cause 5-8% of acute respiratory disease in infants, plus a wide array of other syndromes, including pharyngoconjunctival fever, follicular conjunctivitis, epidemic keratoconjunctivitis, myocarditis, hemorrhagic cystitis, acute diarrhea, intussusception, and encephalomyelitis. Adenoviral pneumonia may have serious long-term sequelae, including bronchiolitis obliterans. Only one third of the 49 serotypes have been associated with disease.

Etiology.

The Adenoviridae are DNA viruses of intermediate size, which are classified into subgenera A to F. The virion has an icosahedral coat (capsid) made up of 252 subunits (capsomers) of which 240 are "hexons" and 12 are "pentons." The hexons have a cross reacting antigen common to all mammalian adenoviruses. The penton confers type specificity, and antibody to it is protective. Adenoviruses can also be classified by their characteristic DNA "fingerprints" on gels after being digested with restriction endonucleases, and this classification generally conforms to their antigenic types.

All adenovirus types, except types 40 and 41, grow in primary human embryonic kidney cells, and most grow in HEp-2 or HeLa cells, producing a typical destructive cytopathic effect. Types 40 and 41 (and other serotypes as well) grow in 293 cells, a line of human embryonic kidney cells into which certain "early" adenovirus genes have been introduced.

Many adenovirus types, but particularly the common childhood types (1, 2, and 5), are shed for prolonged periods from both the respiratory and gastrointestinal tracts. These types also establish low-level and chronic infection of the tonsils and adenoids.

Epidemiology.

Adenoviral infections are distributed worldwide. They occur year-round but are most prevalent in spring or early summer and again in midwinter in temperate climates. Certain types tend to occur in epidemics, notably types 4 and 7 in outbreaks of febrile respiratory disease, types 3, 7, and 21 in severe pneumonia; type 3 in pharyngoconjunctival fever; type 11 in hemorrhagic cystitis; and types 8, 19, and 37 in epidemic keratoconjunctivitis. For unexplained reasons, adenovirus types 3 and 7 cause severe epidemics of pneumonia in the children of northern China and Korea, with mortality rates in hospitalized cases of 5-15%.

Taken from Nelson Textbook of Pediatrics by Behrman.

Page 7: usmle questionnarie

A 45-year-old man presents to the physician because he has been feeling tired and lethargic for

the last 6 months. He states that he has experienced difficulty concentrating and has become

increasingly indecisive during this time. He reports that his wife has told him on numerous

occasions that he snores loudly. He does not take any regular medications and is allergic to

acetylsalicylic acid. He typically drinks 50 to 70 g of ethanol weekly and smokes a half a pack of

tobacco daily. His vital signs are as follows: blood pressure (BP), 140/100 mm Hg; pulse, 59

beats/minute; temperature, 37.5°C (99.5°F); respirations, 12 breaths/minute; and body mass

index (BMI), 35 kg/m2 (35 lb/in2). Physical examination is within normal limits. What is the most

appropriate next step in the management of this patient?

A. Amitriptyline

B. Electroencephalography (EEG)

C. Lorazepam

D. Polysomnography

E. Sertraline

QID: 33526

Option D (Polysomnography) is correct. This patient has daytime sleepiness, fatigue, difficulty concentrating, and significant diastolic hypertension. He is obese and reports loud snoring when he sleeps. This is suggestive of sleep apnea, obstructive type and should be ruled out prior to instituting treatment. The diagnosis can be made using polysomnography.

Option A (Amitriptyline) is incorrect. Amitriptyline is an antidepressant with sedating properties. Appropriate diagnosis should be made prior to any therapy.

Option B (Electroencephalography [EEG]) is incorrect. EEG is used in polysomnography. The key is obtaining an EEG while the patient is sleeping.

Option C (Lorazepam) is incorrect. A benzodiazepine would be contraindicated if this individual had untreated sleep apnea.

Option E (Sertraline) is incorrect. Antidepressant medication would be warranted if there were a diagnosis of major depressive episode. Often, individuals with sleep apnea will present with depressive symptoms.

Obstructive Sleep Apnea

1. Intermittent upper airway obstruction that causes snoring and apneic episodes throughout the night

2. Patients experience decreased rapid eye movement (REM) sleep and wake up tired 3. Evaluation: Overnight sleep study using polysomnography

Page 8: usmle questionnarie

4. Treatment: Continuous positive airway pressure mask, weight reduction in obese persons, surgery

A young patient is transported from the scene of an automobile accident to the ER. The patient

complains of pelvic pain. Radiography of the pelvis is ordered and reveals the image shown (see

figure). Which of the following is the most likely diagnosis?

A. Fracture of pubic symphysis

B. Intra-articular fracture of the left acetabulum

C. Rupture bladder

D. Ruptured urethra

E. Transverse fracture of left femoral head

QID: 27344

Page 9: usmle questionnarie

Option D (Ruptured urethra) is correct. The figure shows abnormal widening of the pubic symphysis with an associated ruptured urethra and a high-riding bladder.

Option A (Fracture of pubic symphysis) is incorrect. There is widening of the pubic symphysis, but no fracture is seen. The figure shows a ruptured urethra and a high-riding bladder.

Option B (Intra-articular fracture of the left acetabulum) is incorrect. This cannot be evaluated with this figure. There is no abnormality of the acetabular and femoral head regions. Abnormal widening of the pubic symphysis with an associated ruptured urethra and a high-riding bladder are seen in this figure.

Option C (Rupture bladder) is incorrect. The bladder is high riding, but there is no evidence of a rupture. Rupture of the urethra is present.

Option E (Transverse fracture of left femoral head) is incorrect. The left femoral head has no evidence of fracture. Abnormal widening of the pubic symphysis with an associated ruptured urethra and a high-riding bladder are seen in this figure.

Urethral Injuries

Urethral injuries are associated with 4% to 14% of all pelvic fractures17,18 and are more common in cases of bilateral pelvic injuries.19,20 Diagnosis of urethral injuries is made by a high index of suspicion in the presence of blood at the urethral meatus, inability to urinate, and/or a palpable full bladder on abdominal examination. When blood is present at the meatus, retrograde urethrography aids in diagnosis of any urethral injury. In the presence of minor urethral injury, a catheter can be placed by an experienced urologist with or without the aid of a cystoscope.21

Urethral injuries are classified as those confined to the posterior urethra (above the urogenital diaphragm) and to the anterior urethra (below the urogenital diaphragm). Posterior urethral injuries are further subclassified as type I (urethral stretch), type II (urethral disruption proximal to the urogenital diaphragm), and type III (proximal and distal disruption of the urogenital diaphragm).

For treatment of posterior urethral injuries, early endoscopic realignment has become more accepted as an excellent initial treatment option.22 Realignment of the damaged urethra with a stented Foley catheter can lead to complete healing of the urethral injury or need for future endoscopic treatment of developed urethral strictures. If realignment of the damaged urethra cannot be achieved, then suprapubic catheterization, followed by delayed combined antegrade and retrograde endoscopic repair or open surgical repair are the potential treatment options.

page 2296

page 2297

    Figure 76-17 Extraperitoneal bladder injury. Contrast agent is extravasated to the space of Retzius (asterisk) after retrograde filling of the bladder through the indwelling Foley catheter.

Page 10: usmle questionnarie

In contrast to posterior urethral injuries that are often associated with many other pelvic injuries, anterior urethral injuries are often isolated and often associated with straddle injuries. The bulbar urethra is often the site of injury. The best initial treatment modality for anterior urethral injuries is not well defined; however, most would agree that primary realignment with Foley catheter, if possible, is the best initial treatment. In cases of severe anterior urethral injury, a suprapubic catheter may be required, followed by delayed open surgical repair.23

Taken from Sabiston Textbook of Surgery by Townsend.

A 63-year-old man presents to the physician, because he has been experiencing difficulty

obtaining and sustaining an erection while being sexually intimate with his wife. He was

diagnosed with type II diabetes 5 years ago and has been poorly compliant with therapy. He also

has a history of benign prostatic hypertrophy and stable angina. His current medications include

metformin, doxazosin, and isosorbide dinitrate. On examination, he has reduced pinprick

sensation bilaterally in the lower extremities. At the end of the consultation, the patient requests

therapy with sildenafil for his erectile dysfunction. What is the most appropriate advice for this

patient with regard to his current therapeutic regimen?

A. Use of sildenafil and doxazosin is contraindicated

B. Use of sildenafil and metformin is contraindicated

C. Use of sildenafil is not recommended within 12 hours of administration of isosorbide dinitrate

D. Use of sildenafil is not recommended within 4 hours of administration of metformin

E. Use of sildenafil is not recommended within 4 hours of administration of doxazosin

QID: 33239

Option E (Use of sildenafil is not recommended within 4 hours of administration of doxazosin) is correct. Sildenafil is a 5c-cyclic guanosine monophosphate (cGMP)-specific phosphodiesterase type 5 inhibitor. This causes results in the enhancement of nitric oxide in the corpus cavernosum. In patients with concurrent benign prostatic hypertrophy, there was a significant decrease in blood pressure when the two drugs were taken within 4 hours of each other. Therefore, it is important to avoid the use of alpha blockers and sildenafil within 4 hours of use.

Option A (Use of sildenafil and doxazosin is contraindicated) is incorrect. Use of these sildenafil and doxazosin is not contraindicated, but rather, must be separated in time because of the potential for hypotension.

Page 11: usmle questionnarie

Option B (Use of sildenafil and metformin is contraindicated) is incorrect. Use of these sildenafil and metformin is not contraindicated, because clinical trials have not demonstrated any adverse events.

Option C (Use of sildenafil is not recommended within 12 hours of administration of isosorbide dinitrate) is incorrect. Sildenafil potentiates nitric oxide. Therefore, sildenafil potentiates the hypotensive effects of nitrates and is therefore completely contraindicated in patients.

Option D (Use of sildenafil is not recommended within 4 hours of administration of metformin) is incorrect. Erectile dysfunction typically develops in patients with diabetes mellitus (DM), often secondary to diabetic neuropathy. Metformin and sildenafil are not contraindicated and should not be separated in time.

PHOSPHODIESTERASE TYPE V INHIBITORS

Sildenafil, the first selective phosphodiesterase type V inhibitor (see also Chs 14, 18), was being developed for another possible indication and was found incidentally to influence erectile function. In contrast to intracavernosal vasodilators, it is not sufficient of itself to cause erection independent of sexual desire, but it enhances the erectile response to sexual stimulation. It has transformed the treatment of erectile dysfunction.

Mechanism of action

Phosphodiesterase V is the isoform that inactivates cGMP. Nitrergic nerves release nitric oxide (or a related nitrosothiol), which diffuses into smooth muscle cells where it activates guanylate cyclase. The resulting increase in cytoplasmic cGMP mediates vasodilation via activation of protein kinase G (Ch. 14). Consequently, inhibition of phosphodiesterase V potentiates the effect on penile vascular smooth muscle of endothelium-derived nitric oxide and of nitrergic nerves that are activated by sexual stimulation. Other vascular beds are also affected, suggesting other possible uses.**

From Pharmacology 5E by Rang et al

Pharmacokinetic aspects and drug interactions

Peak plasma concentrations occur approximately 30-120 minutes after an oral dose and are delayed by eating, so it is taken an hour or more before sexual activity. It is given as a single dose as needed. (For possible long-term indications requiring 24 hour enzyme inhibition, it needs to be given three times daily.) It is metabolised by the 3A4 isoenzyme of cytochrome P450, which is induced by carbamazepine, rifampicin and barbiturates and inhibited by cimetidine, macrolide antibiotics, antifungal imidazolines, some antiviral drugs (such as ritonavir) and also by grapefruit juice (Ch. 8). These drugs can potentially interact with sildenafil in consequence. A dramatic pharmacodynamic interaction occurs with organic nitrates, which work through increasing cGMP (Ch. 17) and are, therefore, markedly potentiated by sildenafil. Consequenty, concurrent nitrate use contraindicates sildenafil.

From Pharmacology 5E by Rang et al

A 51-year-old man is brought into the emergency room after three episodes of vomiting bright red

blood. He has a known history of cirrhosis secondary to alcoholic liver disease. On arrival, his

vital signs are as follows: blood pressure, 90/60 mm Hg; pulse, 115 beats/minute; temperature

37.6°C (99.6°F); and respirations, 18 breaths/minute. His abdomen is dull to percussion

Page 12: usmle questionnarie

throughout. A nasogastric tube is inserted, and bright red blood is found when stomach contents

are aspirated. What is the most appropriate next step in the management of this patient?

A. Balloon tamponade

B. Endoscopic band ligation

C. Intravenous normal saline

D. Octreotide

E. Propranolol

QID: 33275

Option C (Intravenous [IV] normal saline) is correct. This patient has an upper gastrointestinal bleed, most likely the result of bleeding esophageal varices. However, the exact cause is unimportant in this case, because the patient is hemodynamically unstable. He is hypotensive and tachycardic and thus requires hemodynamic resuscitation. The option of IV normal saline meets this requirement.

Option A (Balloon tamponade) is incorrect. Balloon tamponade is reserved for patients who continue to have bleeding esophageal varices despite endoscopic band ligation.

Option B (Endoscopic band ligation) is incorrect. If medical therapy with octreotide fails, endoscopic band ligation is the next step in the management of bleeding esophageal varices.

Option D (Octreotide) is incorrect. Were this patient hemodynamically stable, the use of octreotide is considered the first step in the management of bleeding esophageal varices.

Option E (Propranolol) is incorrect. Propranolol and other nonselective beta blockers are used as prophylaxis against bleeding esophageal varices. There is no role acutely, especially not in hypotensive patients.

APPROACH TO THE PATIENT WITH ACUTE GASTROINTESTINAL BLEEDING (Fig. 32-2)

Assessment of Vital Signs/Resuscitation

The first step in the evaluation and therapy for the patient with acute GI hemorrhage is to determine the severity of blood loss. Vital signs should be recorded immediately. If the systolic blood pressure drops more than 10 mm Hg and/or the pulse increases more than 10 beats per minute as the patient changes positions from supine to standing, it is likely the patient has lost at least 800 mL (15%) of circulating blood volume. Hypotension, tachycardia, tachypnea, and mental status changes in the setting of acute GI hemorrhage suggest at least a 1500-mL (30%) loss of circulating blood volume.

Page 13: usmle questionnarie

The goal of resuscitation is to restore the normal circulatory volume. Initially, at least two large-bore intravenous catheters are used to administer isotonic solutions (e.g., lactated Ringer's solution or 0.9% NaCl), and blood products if indicated. If the patient is in shock, a central venous access should be established. The amount of blood products to be transfused must be individualized. Transfusions of packed red blood cells are provided to prevent complications (e.g., angina, congestive heart failure, stroke) of acute blood loss. Therefore, the need for blood transfusion depends on multiple factors, including the patient's age, overall health, and response of vital signs to initial resuscitation. In view of the risks of blood transfusion, it is not appropriate simply to transfuse until an arbitrary hematocrit is achieved. If coagulation studies are abnormal, as commonly observed in cirrhotic patients, fresh-frozen plasma and/or platelets may be required to control ongoing hemorrhage. Opinions differ regarding the use of nasogastric lavage in preparation for endoscopic examination, although aspiration of gastric blood may be particularly important in patients with liver cirrhosis, because blood in the GI tract may precipitate hepatic encephalopathy.

From Cecil Essentials of Medicine 6E by Andreoli et al

Table 32-3. Common Sources of Acute Gastrointestinal Hemorrhage

Source Associated Clinical Features TreatmentsUpper Gastrointestinal Tract

Esophagitis Heartburn, dysphagia, odynophagia Medication*Antireflux surgery or procedures

Esophageal cancer Progressive dysphagia, weight loss Chemoradiotherapy, surgeryPalliative endoscopy procedures

Gastritis/gastric ulcer Aspirin/NSAID use Withdraw NSAIDs

Duodenitis/duodenal ulcer Abdominal pain/dyspepsia Medication†

Helicobacter pylori infection Endoscopic therapy for acute bleeding

Gastric cancer Early satiety, weight loss, abdominal pain Surgery, chemotherapy

Esophagogastric varices History of CLD Variceal banding, sclerotherapy

Stigmata of CLD on examination Vasopressin, octreotideTIPS or decompressive surgery

Mallory-Weiss tear History of retching before hematemesis Supportive (usually self-limited)Endoscopic therapy

Lower Gastrointestinal Tract

Infection History of exposure, diarrhea, fever Supportive/antibiotics

Inflammatory bowel diseases

History of colitis, diarrhea, abdominal pain, fever

Steroids/5-ASA/immunotherapySurgery if no response to medication

Diverticula Painless hematochezia SupportiveSurgery for recurrent disease

Angiodysplasia Painless hematochezia Endoscopic therapy

Often in ascending colon Supportive

Commonly involves stomach and small bowel as well

Surgery for localized disease

Colon cancer Change in bowel habit, anemia, weight loss Surgery

Colon polyp Usually asymptomatic Endoscopic or surgical removal

Ischemic colitis Typically elderly patientsHistory of vascular diseaseMay present with abdominal pain

Supportive (self-limited)

Meckel's diverticulum Painless hematochezia in young patientLocated at distal ileum

Surgery

Hemorrhoids Rectal bleeding associated with bowel Supportive

Page 14: usmle questionnarie

movement

From Cecil Essentials of Medicine 6E by Andreoli et al

Figure 32-2 Approach to the patient with acute gastrointestinal bleeding. EGD = esophagogastroduodenoscopy.

The medical intern on call is summoned to the bedside of a 70-year-old woman who is

complaining of chest pain. She is postoperative day 2 after a hysterectomy for endometrial

cancer. A 12-lead electrocardiogram (ECG) is ordered, and the tracing is shown (see figure).

Based only on the evidence presented, which of the following is the most likely diagnosis?

A. Acute pericarditis

B. Acute pulmonary embolism

C. Inferior myocardial infarction

D. Left bundle branch block

E. Non–Q-wave myocardial infarction

QID: 27272

Option B (Acute pulmonary embolism) is correct. This is a classic 12-lead ECG indicating an acute pulmonary embolism. There is an S wave in lead I, a Q wave in lead III, and an inverted T wave in lead III (S1, Q3, T3 pattern). There is sinus tachycardia (160 bpm)

Page 15: usmle questionnarie

and an incomplete right bundle branch block pattern (an R wave in aVR and V1 and an S wave in V6).

Option A (Acute pericarditis) is incorrect. The ECG in acute pericarditis generally shows diffuse ST-segment elevation. This is the classic ECG for an acute pulmonary embolism, showing an S wave in lead I, a Q wave in lead III, and an inverted T wave in lead III (S1, Q3, T3 pattern).

Option C (Inferior myocardial infarction) is incorrect. The ECG of an inferior myocardial infarction would show raised ST segment and Q waves in the inferior leads (II, III, and aVF). This is the classic ECG for an acute pulmonary embolism, showing an S wave in lead I, a Q wave in lead III, and an inverted T wave in lead III (S1, Q3, T3 pattern).

Option D (Left bundle branch block) is incorrect. There is an incomplete right bundle branch block pattern (an R wave in aVR and V1 and an S wave in V6). The ECG showing left bundle branch block would typically show QRS duration greater than 0.12 s; broad notched R waves with ST depression in leads I, aVL, and V6; and broad QS waves in V1 to V3. This is the classic ECG for an acute pulmonary embolism, showing an S wave in lead I, a Q wave in lead III and an inverted T wave in lead III (S1, Q3, T3 pattern).

Option E (Non–Q-wave myocardial infarction) is incorrect. The ECG leads corresponding to the infarct region usually show ST depression and deeply inverted symmetrical T waves in non–Q-wave, or subendocardial, infarcts. This is the classic ECG for an acute pulmonary embolism, showing an S wave in lead I, a Q wave in lead III, and an inverted T wave in lead III (S1, Q3, T3 pattern).

ELECTROCARDIOGRAPHY. page 562

page 563

Table 94-3. DETERMINING THE PRETEST PROBABILITY OF ACUTE PULMONARY EMBOLISM BASED ON POINT SYSTEM AND D-DIMER

RESULT

VARIABLE POINTSDVT symptoms/signs* 3.0

PE as or more likely† 3.0

HR >100 beats/min 1.5

Immobilization/surgery‡ 1.5

Previous DVT or PE 1.5

Hemoptysis 1.0

Malignancy 1.0

TOTAL SCORE PRETEST PROBABILITY§

<2.0 Low

2.0 to 6.0 Moderate

>6.0 High

   

*Including objectively measured leg swelling and pain with palpation in the deep vein region.†PE as likely or more likely than an alternative diagnosis. Physicians were told to use clinical information, along with chest radiography, electrocardiography, and laboratory tests.‡If in previous 4 weeks.§Of the 437 patients with a negative D-dimer result (by the SimpliRED assay) and low clinical probability, only one developed PE during follow-up; thus, the negative predictive value for the combined strategy of using the clinical model with D-dimer testing in these patients was 99.5%.DVT =deep venous thrombosis, PE =pulmonary embolism, HR =heart rate.From Wells PS, Anderson DR, Rodger M, et al: Excluding pulmonary embolism at the bedside without diagnostic imaging: Management of patients with suspected pulmonary embolism presenting to the emergency department by using a simple clinical model and D-dimer. Ann Intern Med 2001;135:98.

Page 16: usmle questionnarie

Electrocardiographic findings, which are present in the majority of patients with acute PE, include ST-segment abnormalities, T-wave changes, and left or right axis deviation. Only one third of patients with massive or submassive emboli have manifestations of acute cor pulmonale such as the S1-Q3-T3 pattern, right bundle branch block, P-wave pulmonale, or right axis deviation. All of these findings are also nonspecific. The utility of electrocardiography in suspected acute PE is derived more from its ability to establish or exclude alternative diagnoses, such as acute myocardial infarction, rather than diagnosing or excluding PE.

CHEST RADIOGRAPHY.

    Figure 94-2 High probability ventilation-perfusion scan.

The chest radiograph is often abnormal in patients with acute PE, but it is nearly always nonspecific. Common radiographic findings include pleural effusion, atelectasis, pulmonary infiltrates, and mild elevation of a hemidiaphragm. Classic findings of pulmonary infarction, such as Hampton's hump or decreased vascularity (Westermark's sign), are suggestive of the diagnosis, but they are infrequent. A normal chest radiograph in the setting of dyspnea and hypoxemia without evidence of bronchospasm or anatomic cardiac shunt is strongly suggestive of PE. Under most circumstances, however, the chest radiograph cannot be used for conclusive diagnosis or exclusion. Although the radiograph may exclude other processes, such as pneumonia, pneumothorax, or rib fracture, which may cause symptoms similar to acute PE, PE may frequently coexist with other underlying heart or lung diseases. Symptoms, signs, radiographic findings, electrocardiography, and the plasma D-dimer measurement cannot be considered diagnostic of PE or DVT. When these entities are suspected, further evaluation with noninvasive or invasive testing is necessary.

Taken from Cecil Textbook of Medicine by Goldman.

A 27-year-old woman develops the pictured lesion 2 days after wearing a new pair of earrings.

What is the most likely diagnosis?

Page 17: usmle questionnarie

A. Allergic contact dermatitis

B. Atopic dermatitis

C. Irritant contact dermatitis

D. Nummular dermatitis

E. Seborrheic dermatitis

QID: 33610

Option A (Allergic contact dermatitis) is correct. This patient has allergic contact dermatitis, most likely the result of nickel in her new earrings. Allergic contact dermatitis should be suspected when there is exposure to an allergen and a reaction develops at least 48 hours later. As pictured, there is an erythematous base and a slightly white scale with a defined border.

Option B (Atopic dermatitis) is incorrect. Atopic dermatitis is a subacute and chronic dermatitis that is often called an itch that rashes. It has dry, scaly, pruritic patches and plaques with excoriations located in the flexural regions.

Option C (Irritant contact dermatitis) is incorrect. Irritant contact dermatitis is the result of direct toxic injury to the skin and will occur in any individual given sufficient exposure. This is in contrast to allergic contact dermatitis, where individuals with atopy are more likely to develop it. Irritant contact dermatitis is differentiated from allergic contact dermatitis by the acute speed of the reaction (less than 12 hours usually), a very sharp border without spread and an absence of papules in the acute phase.

Option D (Nummular dermatitis) is incorrect. Nummular dermatitis presents as pruritic, coin-shaped erythematous plaques that are dry and scaly.

Option E (Seborrheic dermatitis) is incorrect. Seborrheic dermatitis presents with a greasy, yellow, erythematous, scaly plaque primarily in the perioral area or other areas rich in sebaceous glands, such as the scalp margin and sternum.

Dermatitis, Contact (PTG)

BASIC INFORMATION

DEFINITION

Contact dermatitis is an acute or chronic skin inflammation, usually eczematous dermatitis resulting from exposure to substances in the environment. It can be subdivided into "irritant" contact dermatitis (nonimmunologic physical and chemical alteration of the epidermis) and "allergic" contact dermatitis (delayed hypersensitivity reaction).

From Ferri's Clinical Advisor 2006 by Ferri

Page 18: usmle questionnarie

PHYSICAL FINDINGS & CLINICAL PRESENTATION

IRRITANT CONTACT DERMATITIS:

Mild exposure may result in dryness, erythema, and fissuring of the affected area (e.g., hand involvement in irritant dermatitis caused by exposure to soap, genital area involvement in irritant dermatitis caused by prolonged exposure to wet diapers).

Eczematous inflammation may result from chronic exposure.

ALLERGIC CONTACT DERMATITIS:

Poison ivy dermatitis can present with vesicles and blisters; linear lesions (as a result of dragging of the resins over the surface of the skin by scratching) are a classic presentation.

The pattern of lesions is asymmetric; itching, burning, and stinging may be present. The involved areas are erythematous, warm to touch, swollen, and may be confused

with cellulitis.

ETIOLOGY

Irritant contact dermatitis: cement (construction workers), rubber, ragweed, malathion (farmers), orange and lemon peels (chefs, bartenders), hair tints, shampoos (beauticians), rubber gloves (medical, surgical personnel)

Allergic contact dermatitis: poison ivy, poison oak, poison sumac, rubber (shoe dermatitis), nickel (jewelry), balsam of Peru (hand and face dermatitis), neomycin, formaldehyde (cosmetics)

From Ferri's Clinical Advisor 2006 by Ferri

DIAGNOSIS

DIFFERENTIAL DIAGNOSIS

Impetigo Lichen simplex chronicus Atopic dermatitis Nummular eczema Seborrheic dermatitis Psoriasis Scabies

WORKUP

Medical history: gradual onset vs. rapid onset, number of exposures, clinical presentation, occupational history

Physical examination: contact dermatitis in the neck may be caused by necklaces, perfumes, after-shave lotion; involvement of the axillae is often secondary to deodorants, clothing; face involvement can occur with cosmetics, airborne allergens,

Page 19: usmle questionnarie

aftershave lotion

LABORATORY TESTS

Patch testing is useful to confirm the diagnosis of contact dermatitis; it is indicated particularly when inflammation persists despite appropriate topical therapy and avoidance of suspected causative agent; patch testing should not be used for irritant contact dermatitis because this is a nonimmunologic-mediated inflammatory reaction.

Gram stain and cultures are indicated only in cases of suspected secondary infection or impetigo.

From Ferri's Clinical Advisor 2006 by FerriA 52-year-old woman visits her primary care physician for a regularly scheduled check-up. She

has no current complaints or history of serious illness. There are no abnormal findings on

physical exam or any significant change from her last visit 1 year ago. A routine CBC is ordered

and shows a WBC of 45,000 cells/mm3. The peripheral blood smear shows leukoerythroblastosis

with many myeloblasts, myelocytes, metamyelocytes, and nucleated red blood cells present. A

bone marrow aspirate is obtained and shows a similar microscopic appearance. Cytogenetic

studies of the immature cells are undertaken and show a 9;22 chromosomal translocation. Which

of the following is appropriate treatment at this time?

A. All-trans-retinoic acid

B. Bone marrow transplantation

C. Imatinib

D. Mitoxantrone, vincristine, prednisone, and methotrexate

E. Splenectomy

QID: 27241

Option C (Imatinib) is correct. The cellular appearance of the peripheral smear and bone marrow aspirate is characteristic of chronic myelogenous leukemia. The presence of the Philadelphia chromosome (t9;22) make this diagnosis virtually certain. Protein-tyrosine kinase inhibitors such as STI571 or imatinib mesylate that inhibit the bcr-abl tyrosine kinase have revolutionized the treatment of chronic myelogenous leukemia. Recent studies show imatinib is superior to interferon alpha plus low-dose cytarabine as first-line therapy in newly diagnosed, chronic-phase CML.

Option A (All-trans-retinoic acid) is incorrect. Transretinoic acid can effectively induce most newly diagnosed acute promyelocytic leukemia patients into remission, without the myelosuppressive effects of chemotherapy. The cellular appearance of the peripheral smear and bone marrow aspire are characteristic for chronic myelogenous leukemia. The presence

Page 20: usmle questionnarie

of the Philadelphia chromosome (t9;22) make this diagnosis virtually certain. This is not the treatment for CML.

Option B (Bone marrow transplantation) is incorrect. Bone marrow transplantation after 1 year of treatment without complete or significant cytogenetic remission is recommended in patients 55 years of age or younger with CML and with a sibling donor matched for HLA-A, HLA-B, and HLA-DR.

Option D (Mitoxantrone, vincristine, prednisone, and methotrexate) is incorrect. This was an experimental induction combination for acute lymphoblastic leukemia and a poor one at that. A cancer and leukemia Group B (CALGB) study of this regimen closed early because the median remission duration was shorter than in previous studies.

Option E (Splenectomy) is incorrect. Splenectomy offers no intrinsic benefit in the treatment of CML and is not indicated in patients whose disease is well controlled. Evidence exists suggesting splenectomy may accelerate the onset of myeloid metaplasia in the liver. Splenectomy is associated with high perioperative rates of bleeding or thrombotic complications.

Imatinib

Imatinib is a 2-phenylaminopyrimidine agent selectively inhibiting the c-abl tyrosine kinase. The development of imatinib over 14 years is likely to become a paradigm for cancer therapeutic development in the future.

Chronic myelogenous leukemia (CML) has a characteristic chromosomal translocation, 9:22. This places the cellular homologue of the feline Abelson leukemia virus tyrosine kinase downstream from the break point cluster region, bcr, and results in the unregulated expression of the fusion p210Bcr-Abl oncogene, which functions as a cytoplasmic protein kinase. Bcr-Abl is necessary and sufficient to produce CML, although additional chromosomal aberrations develop in the later stages of the disease. Imatinib interferes with the binding of ATP to the tyrosine kinase site on abl. Imatinib produces hematologic remission in virtually 100% of interferon-refractory patients with the accelerated phase of CML. Nearly 30% of patients have disappearance of the Philadelphia chromosome/9:22 translocation. Imatinib has activity against two tyrosine receptor kinases, c-kit and PGDF (platelet derived growth factor). C-kit mutations occur in 70% of patients with gastrointestinal stromal tumors (GIST, a sarcoma arising from the myenteric neurons of Cajal). Imatinib produces significant responses in this otherwise refractory sarcoma.

Taken from Integrated Pharmacology 2E by Page et al

TREATMENT

1. Imatinib mesylate (Gleevec), an oral tyrosine kinase inhibitor, is effective and indicated as first-line treatment for CML myeloid blast crisis, accelerated phase, or CML in its chronic phase. More than 60% of patients have major cytogenetic response (<35% Philadelphia chromosome-positive cells in the marrow) and more than 80% have progression-free survival after 24 months. Complete hematologic response usually occurs in less than 1 month.

2. Symptomatic hyperleukocytosis (e.g., CNS symptoms) can be treated with leukapheresis and hydroxyurea; allopurinol should also be started to prevent urate nephropathy following rapid lysis of the leukemic cells.

3. Cytotoxic therapy with hydroxyurea has largely replaced busulfan as the standard

Page 21: usmle questionnarie

cytotoxic treatment. 4. Allogeneic stem-cell transplantation (following intense chemotherapy with busulfan

and cyclophosphamide or combined chemotherapy with cyclophosphamide and fractionated total body irradiation to destroy residual leukemic cells) is a potentially curative treatment for CML in chronic phase unresponsive to imatinib. Generally only 20% of patients are candidates for stem-cell transplantation, given the limitations of age and lack of HLA-matched donors.

a. It should be considered in "young" patients (increased survival in patients younger than 55 years) with compatible siblings.

b. Early transplantation is also very important for patient survival. 5. Transplantation of marrow from an HLA-matched, unrelated donor is also now

recognized as safe and effective therapy for selected patients.

Taken from Practical Guide to the Care of the Medical Patient 6E by Ferri

A 22-year-old G1P0 at 34 weeks’ gestation is involved in a motor vehicle accident. She was not

wearing her seat belt and as she jolted forward, her abdomen hit the steering wheel. She is

brought to labor and delivery for assessment. She is observed for 2 hours. The maternal status is

stable. Fetal heart trace is reassuring. No uterine contractions are noted on external uterine

monitoring. There is no evidence of vaginal bleeding. The ultrasound examination does not

detect a placental abruption. Which of the following is the most appropriate next step in patient

care?

A. Administer betamethasone

B. Deliver by cesarean section

C. Discharge home on bed rest

D. Induce labor

E. Obtain a blood type and Rh status

QID: 26721

Option E (Obtain a blood type and rhesus [Rh] status) is correct. A pregnant woman involved in a motor vehicle accident in which abdominal trauma has occurred should have blood type and Rh status checked. Those women who are Rh-negative should be given RhoGAM.

Option A (Administer betamethasone) is incorrect. Betamethasone is not indicated at this point because there is no indication of preterm delivery.

Option B (Deliver by cesarean section) is incorrect. There is no indication to deliver this

Page 22: usmle questionnarie

patient in that both she and the fetus are stable.

Option C (Discharge home on bed rest) is incorrect. Before being discharged home, the patient should have her blood type and Rh status checked because those women who are Rh-negative should receive RhoGAM.

Option D (Induce labor) is incorrect. There is no indication to deliver this patient in that both she and the fetus are stable.

PREVENTION OF RHESUS ISOIMMUNIZATION

Because Rh isoimmunization occurs in response to exposure of an Rh-negative mother to the Rh antigen, the mainstay for prevention is the avoidance of maternal exposure to the antigen. RhO-GAM diminishes the availability of the Rh antigen to the maternal immune system, although the exact mechanism by which it prevents Rh isoimmunization is not well understood.

RhO-GAM is prepared from fractionated human plasma obtained from hyperreactive sensitized donors. The plasma is screened for hepatitis B surface antigen and anti-HIV-1, the antibody to the acquired immunodeficiency syndrome (AIDS) virus. The globulin is available in several dosages for intramuscular injection. Since the advent of its use in 1967, Rh immune globulin has dramatically reduced the incidence of Rh isoimmunization.

Because the greatest risk for fetal-to-maternal hemorrhage occurs during labor and delivery, Rh immune globulin was initially administered only during the immediate postpartum period. This resulted in a 1% to 2% failure rate, thought to be due to exposure of the mother to fetal red blood cells during the antepartum period. The indications for the use of Rh immune globulin have therefore been broadened to include any antepartum event (such as amniocentesis) that may increase the risk of transplacental hemorrhage. The routine prophylactic administration of Rh immune globulin at 28 weeks' gestation is now the standard of care. Despite adherence to this suggested Rh immune globulin protocol, 0.27% of primiparous Rh-negative patients still become sensitized.

Taken from Essentials of Obstetrics & Gynecology 4E by Hacker et al

INDICATIONS FOR ADMINISTRATION OF RHO-GAM

The following provides a practical approach to the administration of Rh immune globulin to an Rh-negative patient with no Rh antibodies.

During a normal pregnancy, 300 μg of Rh immune globulin is administered at 28 weeks' gestation, following testing for sensitization with an indirect Coombs' test. A 300-μg dose is administered following amniocentesis at any gestational age. If a fetomaternal hemorrhage is suspected at any time during the pregnancy, a Kleihauer-Betke test should be performed. If positive, Rh immune globulin is administered in a dose of 10 μg/mL of fetal blood that entered the maternal circulation. Following an uncomplicated delivery, 300 μg of Rh immune globulin is given within 72 hours. If a larger than normal fetal-to-maternal hemorrhage is suspected, such as may occur in patients with abruptio placentae or those requiring cesarean section or manual removal of the placenta, a Kleihauer-Betke determination should be performed after delivery and the appropriate dose of the Rh immune globulin determined.

Page 23: usmle questionnarie

Establishment of fetal circulation occurs at approximately 4 weeks' gestation, and the presence of the RhO D antigen has been demonstrated as early as 38 days following conception. Consequently, Rh isoimmunization can occur at any time during pregnancy, from the early first trimester on. Because fetal erythrocytes can be readily detected in the maternal blood following induced or spontaneous abortion, 50 μg of Rh immune globulin should be given to all Rh-negative women following any type of abortion.

Fetal erythrocytes have been demonstrated in the maternal circulation following rupture of a tubal pregnancy. Consequently, Rh immune globulin should be given to an Rh-negative woman with an ectopic pregnancy. Because chorionic villi in gestational trophoblastic disease are avascular and are devoid of fetal erythrocytes, Rh immune globulin is probably not necessary following molar pregnancy. At least one case of sensitization following a molar pregnancy, however, has been reported.

Taken from Essentials of Obstetrics & Gynecology 4E by Hacker et al

A 34-year-old gravida IV para III female at 37 weeks of gestation is brought to the emergency

room following the sudden onset of abdominal pain and vaginal bleeding 2 hours ago. The pain

has been constant from onset. She has also experienced frequent, strong contractions. She has

been otherwise well, abstinent from intercourse for the past 2 months, and her previous medical

and obstetric history are unremarkable. Her vital signs are blood pressure (BP), 100/65 mm Hg;

pulse, 110 beats/minute; temperature, 37.7°C (99.8°F); and respirations, 14 breaths/minute. The

uterus is painful and rigid. Speculum examination demonstrates mild vaginal bleeding. Fetal

heart rate tracing is not reassuring. What is the most likely diagnosis?

A. Abruptio placenta

B. Placenta previa

C. Spontaneous abortion

D. Uterine rupture

E. Vasa previa

QID: 33070

Option A (Abruptio placenta) is correct. This patient is demonstrating the classic signs of placental abruption: painful, abrupt vaginal bleeding associated with uterine contractions and nonreassuring fetal heart tracing. The pain is constant, and the uterus is firm because of tetanic contractions. Vaginal bleeding can be highly variable and does not correlate well with the severity of abruption.

Option B (Placenta previa) is incorrect. Placenta previa is classically bright red painless bleeding that is not as rapid in onset as abruption. The uterus is usually soft.

Page 24: usmle questionnarie

Option C (Spontaneous abortion) is incorrect. Spontaneous abortion is defined as a pregnancy that has ended spontaneously before 20 to 22 weeks. At 37 weeks, this fetus is viable.

Option D (Uterine rupture) is incorrect. Uterine rupture is painful third trimester bleeding, but is very rare. Unlike abruptio placenta, it is associated with constant heavy vaginal bleeding. A classic presentation on the examination is a fetus that is at a +2 station that suddenly retracts into the -1 position.

Option E (Vasa previa) is incorrect. Vasa previa has a classic triad of membrane rupture followed by vaginal bleeding and then fetal bradycardia. Vasa previa is bleeding from fetal vessels and is diagnosed using the Apt test.

ABRUPTIO PLACENTAE

Abruptio placentae, or premature separation of the normally implanted placenta, complicates 0.5% to 1.5% of all pregnancies (1 in 120 births). Abruption severe enough to result in fetal death occurs in 1 per 500 deliveries.

From Essentials of Obstetrics & Gynecology 4E by Hacker et al

DIAGNOSIS AND MANAGEMENT

Clinically, the diagnosis of a placental abruption is entertained if a patient presents with painful vaginal bleeding in association with uterine tenderness, hyperactivity, and increased tone. The signs and symptoms of placental abruption are, however, variable. The most common finding is vaginal bleeding, seen in 80% of cases. Abdominal pain and uterine tenderness are seen in 66% of cases, fetal distress in 60%, uterine hyperactivity and increased uterine tone in 34%, and fetal demise in 15%.

Box 11-2. Risk factors for abruptio placentae Maternal hypertension Placental abruption in a prior pregnancy Trauma Polyhydramnios with rapid decompression Premature rupture of membranes Short umbilical cord Tobacco use Folate deficiency

The diagnosis of placental abruption is primarily a clinical one. Ultrasonography may detect only 2% of abruptions. Because placental abruption may coexist with a placenta previa, the reason for doing an initial ultrasonic examination is to exclude the latter diagnosis.

Management of the patient with an abruption includes careful maternal hemodynamic monitoring, fetal monitoring, serial evaluation of the hematocrit and coagulation profile, and delivery. Intensive monitoring of both the mother and the fetus is essential because rapid deterioration of either one's condition can occur. Blood products for replacement should always be available, and a large-bore (16- to 18-gauge) intravenous line must be secured. Red blood cells should be given liberally if indicated.

Page 25: usmle questionnarie

From Essentials of Obstetrics & Gynecology 4E by Hacker et al

A 55-year-old female is brought to the emergency room with a sudden onset of severe chest,

back, and abdominal pain that began 20 minutes ago. The pain is described as being very sharp

with a tearing-like character. Her previous medical history is notable for hypertension treated with

hydrochlorothiazide. She does not smoke tobacco or drink alcohol.

On examination, her blood pressure is 145/95 mm Hg in the right arm and 119/75 mm Hg in the

left arm; pulse, 105 beats/minute; temperature, 37.3°C (99.1°F); and respirations, 17

breaths/minute. A chest X-ray (CXR) demonstrates a widened mediastinum, and

electrocardiography (ECG) reveals nonspecific ST and T wave changes. What is the most

appropriate next step in the management of this patient?

A. Aortography

B. Emergent surgery

C. Intravenous labetalol

D. Thoracic spiral computed tomography (CT)

E. Transesophageal echocardiography (TEE)

QID: 33208

Option C (Intravenous labetalol) is correct. Ripping chest pain that radiates into the back and stomach, combined with a pulse deficit, previous history of hypertension and widened mediastinum on chest X-ray (CXR) should be considered an aortic dissection until proven otherwise. As soon as aortic dissection is suspected, immediate intravenous (IV) beta-blocker therapy should be started to reduce the pulse and blood pressure of the patient in an attempt to limit the severity of the dissection.

Option A (Aortography) is incorrect. Aortography is rarely performed, because it is invasive and requires contrast media into the aorta.

Option B (Emergent surgery) is incorrect. Emergent surgery is indicated in cases of aortic dissection where the proximal aorta is involved. The exact location is determined using computed tomography (CT) scanning and therefore, surgery would not be the next step in the management of this patient.

Option D (Thoracic spiral computed tomography [CT]) is incorrect. Spiral CT scanning is excellent for making the diagnosis of aortic dissection. However, the patient can die in the CT scanner without IV beta-blocker therapy.

Option E (Transesophageal echocardiography [TEE]) is incorrect. TEE is very useful in

Page 26: usmle questionnarie

the diagnosis of aortic dissection and is the preferred imaging modality in very unstable patients. This patient is relatively stable, however and requires beta-blocker therapy.

Chest x-ray may show widened mediastinum (62%) and displacement of aortic intimal calcium.

Transesophageal echocardiography (TEE) is study of choice in unstable patients, but operator dependent.

MRI has the highest sensitivity and specificity but limited availability; not suitable for unstable patients; contraindicated with pacemakers, metal devices.

Helical CT is least operator dependent, involves intravenous contrast. TEE, MRI, helical CT are imaging modalities of choice. Sensitivities (98% to 100%)

and specificities (95% to 98%) nearly equal in skilled hands. Test of choice depends on clinical circumstances and availability.

With medium or high pretest probability, a second diagnostic test should be done if the first is negative.

Transthoracic echocardiography has poor sensitivity Aortography rarely done now.

TREATMENT

ACUTE GENERAL Rx

Admit to ICU for monitoring. Propanolol 1 mg every 3-5 min or metoprolol 5 mg IV every 5 min, followed by

nitroprusside 0.3-10 mg/kg/min, with target SBP 100-120. IV beta-blocker is cornerstone of treatment. IV Labetalol can be used instead; 20 mg IV, then 40-80 mg every 10 min. IV calcium channel blockers with negative inotropy may be used. Multiple medications may be needed. Proximal dissections require emergent surgery to prevent rupture or pericardial

effusion. Distal dissections are treated medically unless distal organ involvement or impending

rupture occurs. Evolving role for endovascular stent placement as less invasive treatment for high-

risk surgical patients.

From Ferri's Clinical Advisor 2008 by Ferri

A 3260-g (7 lb 2 oz) male infant is born at 38 weeks’ gestation to a healthy, 26-year-old woman.

The only abnormality noted during the course of the pregnancy was that the woman’s fundal

height measured consistently “large for date” of gestation. The infant is delivered via

spontaneous, vaginal delivery and has Apgar scores of 9 and 10 at one and five minutes,

respectively. On the first day postpartum, the mother calls for help as she is feeding the infant;

the infant ejects formula from the nose and mouth and then begins to cough, choke, and turn

blue as soon as the mother begins to feed him. Which of the following is the most likely reason

for the mother’s abnormal uterine fundal height during the pregnancy?

A. Abnormal esophageal development, inhibiting the fetus from swallowing amniotic fluid

Page 27: usmle questionnarie

B. Abnormal renal development, causing oligohydramnios

C. Abnormal renal development, causing polyhydramnios

D. Fetal macrosomia, which accounted for large uterine fundal size

E. Hyperglycemia, causing the fetus to produce more urine than normal

QID: 27597

Option A (Abnormal esophageal development, inhibiting the fetus from swallowing amniotic fluid) is correct. The history of large fundal height and choking on first feeding is consistent with tracheoesophageal fistula. Inability of the fetus to swallow amniotic fluid results in polyhydramnios and large fundal height, and the opening between the esophagus and trachea results in choking and aspiration on feeding.

Option B (Abnormal renal development, causing oligohydramnios) is incorrect. Oligohydramnios would cause a smaller than expected fundal height.

Option C (Abnormal renal development, causing polyhydramnios) is incorrect. Normally, maldevelopment of the kidneys causes oligohydramnios and results in a smaller uterine fundal height. Also, a renal problem alone would not explain the choking while feeding.

Option D (Fetal macrosomia, which accounted for large uterine fundal size) is incorrect. At 3260 g (7 lb 2 oz), this infant is not considered macrosomic and would not have been so as a fetus.

Option E (Hyperglycemia, causing the fetus to produce more urine than normal) is incorrect. Maternal hyperglycemia does cause polyuria in the fetus, much as it does in diabetic individuals. However, the question stem states that the mother is healthy. Also, a fetus continually exposed to high blood glucose levels is likely to develop either macrosomia or intrauterine growth restriction.

Examination of the pregnant uterus is vital. The fundal height is measured in centimeters from the symphysis pubis to the top of the uterus. Serial measurements over the course of pregnancy provide an excellent assessment of fetal growth with a rough approximation between centimeters and weeks, gestation from 18-34 weeks in a patient with a normal body habitus. Fetal heart tones should be auscultated, and fetal position and estimated weight determined during the last trimester. Lastly, uterine contractions can be easily palpated during a routine exam and, if frequent and many weeks before term, may point to the possibility of preterm labor.

Taken from Ob/Gyn Secrets 3E by Bader

Page 28: usmle questionnarie

The neonate with EA typically has frothing and bubbling at the mouth and nose, episodes of coughing, cyanosis, and respiratory distress. Feeding exacerbates these symptoms, causes regurgitation, and may precipitate aspiration. Aspiration of gastric contents via a distal fistula causes more damaging pneumonitis than aspiration of pharyngeal secretions from the blind upper pouch. The infant with an isolated TEF in the absence of EA ("H-type" fistula) may come to medical attention later in life with chronic respiratory problems, including refractory bronchospasm and recurrent pneumonias.

Diagnosis.

In the setting of early-onset respiratory distress, the inability to pass a nasogastric or orogastric tube in the newborn is suggestive of esophageal atresia. Prenatally, maternal polyhydramnios may alert the physician to EA. Plain radiography in the evaluation of respiratory distress may reveal a coiled feeding tube in the esophageal pouch and/or an air- distended stomach indicating the presence of a coexisting TEF (Fig. 300-2). Conversely, pure EA may present as an airless, scaphoid abdomen. In isolated TEF, an esophagogram with contrast medium injected under pressure may demonstrate the defect. Alternatively, the orifice may be detected at bronchoscopy or when methylene blue dye injected into the endotracheal tube during endoscopy is observed in the esophagus during forced inspiration.

Management.

    Figure 300-1 Diagrams of the five most commonly encountered forms of esophageal atresia and tracheoesophageal fistula, shown in order of frequency.

Initially, maintaining a patent airway and preventing aspiration of secretions is paramount. Prone positioning minimizes movement of gastric secretions into a distal fistula, and esophageal suctioning minimizes aspiration from a blind pouch. Endotracheal intubation is avoided because it may worsen distention of abdominal viscera. Surgical ligation of the TEF and primary end-to-end anastomosis of the esophagus are performed when feasible. In the premature or otherwise complicated infant, a primary closure may be delayed by temporizing with fistula ligation and gastrostomy tube placement. If the gap between the atretic ends of the esophagus is greater than 3-4 cm, primary repair cannot be done; options include using gastric, jejunal, or colonic segments interposed as a neo-esophagus. Careful search must be undertaken for the common associated cardiac and other anomalies.

   

Figure 300-2 Tracheoesophageal fistula. Lateral radiograph demonstrating a nasogastric tube coiled (arrows) in the proximal segment of an atretic esophagus. The distal fistula is suggested by gaseous dilatation of the stomach (S) and small intestine. The arrowhead depicts vertebral fusion,

whereas a heart murmur and cardiomegaly suggest the presence of a ventricular septal defect. This patient demonstrated elements of the VATER anomalad. (From Balfe D, Ling D, Siegel M: The esophagus. In Putman CE, Ravin CE [editors]: Textbook of Diagnostic Imaging. Philadelphia, WB

Saunders, 1988.)

Taken from Nelson Textbook of Pediatrics by Behrman.

Page 29: usmle questionnarie

A 72-year-old man with a history of smoking 60 packs per year comes to the emergency room

with increased shortness of breath. He usually can increase his oxygen and take more frequent

puffs on his metered-dose inhalers, but this has not helped him on this occasion. His oxygen

saturation is 90% on 3 L. He has decreased breath sounds, which are more obvious on the left

side. The remainder of his examination is only notable for tachycardia. A radiograph of the chest

is shown (see figure). Which of the following is the most appropriate next step in management?

A. 100% O2 via face mask

B. CT scan of the thorax

C. IV Methylprednisolone, Albuterol/Atrovent inhalers, and O2

D. Sputum cultures

QID: 27435

Option A (100% O2 via face mask) is correct. The first treatment of pneumothorax is 100% oxygen via face mask.

Option B (CT scan of the thorax) is incorrect. A CT scan of the thorax may be useful to determine the degree of lung collapse, but the first treatment strategy would be administration of oxygen.

Option C (IV Methylprednisolone, Albuterol/Atrovent inhalers, and O2) is incorrect.

Page 30: usmle questionnarie

This would be the correct management of solely a COPD exacerbation; the pneumothorax demands oxygen via face mask.

Option D (Sputum cultures) is incorrect. Sputum cultures are not indicated in this case; the initial treatment of pneumothorax is administration of oxygen.

Supplemental oxygen increases the rate of pneumothorax absorption. Cautious observation in the asymptomatic patient with <15% pneumothorax can be

done but requires close daily outpatient monitoring.

Taken from Ferri's Clinical Advisor 2006 by Ferri

A 32-year-old man presents to the physician, because he noticed a large amount of blood in the

toilet bowl whist defecating this morning. The blood was bright red in color and located primarily

in the bowl. The stool was brown-colored and did not have an unusual smell. He denies any

abdominal pain, nausea, vomiting, or passage of mucus. Two years ago, he experienced an

isolated episode of vomiting bright red blood. He is currently employed as an aircraft mechanic,

smokes 1 pack of cigarettes a day for the last 10 years, and drinks 30 to 40 g of ethanol weekly.

His father and brother have also had several episodes of bright red blood per rectum, but he

denies any knowledge of a family history of colon cancer. On examination, his vital signs are as

follows: blood pressure, 118/75 mm Hg; pulse, 77 beats/minute; temperature, 37.6°C (99.7°F);

and respirations, 13 breaths/minute. There are multiple telangiectasias on the face, inside the

nose, on the oral mucosa, and upper thorax. The abdomen is tympanic to percussion, and

normal bowel sounds are auscultated. It is soft and nontender to palpation, and there is no

organomegaly or masses. Rectal examination reveals a normal anal reflex and bright red blood

on the glove. There is no mucus or palpable hemorrhoids. What is the most likely diagnosis?

A. Familial colonic polyposis syndrome

B. Gardner syndrome

C. Meckel diverticulum

D. Osler-Rendu-Weber syndrome

E. Peutz-Jeghers syndrome

QID: 33268

Option D (Osler-Rendu-Weber syndrome) is correct. Bright red PR bleeding that is associated with multiple telangiectasias on the skin, lungs, brain, or mucosa combined with a family history is strongly suggestive of Osler-Rendu-Weber syndrome. Bleeding results from rupture of the telangiectasias in the gastrointestinal (GI) tract. Therefore, treatment is with

Page 31: usmle questionnarie

endoscopic injection therapy.

Option A (Familial colonic polyposis syndrome) is incorrect. Also known as polyposis coli, this familial polyposis syndrome results in thousands of adenomatous polyps and carries a 100% risk of developing colon cancer if prophylactic total colectomy is not performed.

Option B (Gardner syndrome) is incorrect. Gardner syndrome is a constellation of adenomatous polyps (therefore, they have malignant potential), soft tissue, and bony tumors.

Option C (Meckel diverticulum) is incorrect. Meckel diverticulum also presents with painless bright red rectal bleeding. However, this diagnosis is less likely, because of the family history of per rectum (PR) bleeding and the numerous telangiectasias found on examination. Meckel diverticulum would be diagnosed with a technetium-uptake scan (also called a Meckel scan).

Option E (Peutz-Jeghers syndrome) is incorrect. Peutz-Jeghers syndrome is one of the familial polyposis syndromes but fortunately produces only hamartomas polyps, which do not have malignant potential. The polyps are found throughout the large and small intestine. There is typically mucocutaneous pigmentation, most commonly around the lips. These pigmentations are not telangiectasias.

7. Is examination of the skin helpful in identifying the source of an upper GI bleed?

The skin examination can be helpful for suggesting a potential source if certain stigmata are present. Lymphadenopathy or abdominal masses may suggest sources for intra-abdominal pathology (Table 5-1).

Table 5-1. SKIN FINDINGS IN CONDITIONS THAT CAUSE GI BLEEDING

Disease Associated skin findingsPeutz-Jeghers Pigmented macules on lips, palms, soles

Malignant melanoma Melanoma

Hereditary hemorrhagic telangiectasias Telangiectasias on lips, mouth, palms, soles (Osler-Weber-Rendu)

Blue rubber bleb nevus Dark, blue soft nodules

Bullous pemphigoid Oral and skin bullae

Neurofibromatosis Café-au-lait spots, axillary freckles, neurofibromas

Cronkhite-Canada Alopecia; hyperpigmentation of creases, hands, and face

Cirrhosis Spider angiomata, Dupuytren's contracture

Neoplasm Acanthosis nigricans

Kaposi's sarcoma Cutaneous Kaposi's sarcoma

Ehlers-Danlos Skin fragility, keloids, paper thin scars

Pseudoxanthoma elasticum Yellow "chicken fat" papules and plaques in flexural areas

Turner's Webbing of neck, purpura, skin nodules

A 47-year-old man presents to the emergency room 30 minutes after the sudden onset of chest

pain is located more prominently over the left hemithorax and does not radiate elsewhere. It is

sharp in quality, notably more severe on inspiration, and does not change with position. He has

been previously well and is not on any current medications. His blood pressure is 130/85 mm Hg;

pulse, 107 beats/minute; temperature, 38.6°C (101.4°F); and respirations, 24 breaths/minute.

Auscultation reveals a fourth heart sound and rales over the left posterior hemithorax.

Electrocardiography (ECG) demonstrates a right bundle branch block that was not present on his

Page 32: usmle questionnarie

previous electrocardiogram performed 1 month ago. What is the most appropriate next step in

the management of this patient?

A. Echocardiography

B. Leg ultrasonography

C. Nebulized salbutamol

D. Pulmonary arteriography

E. Ventilation-perfusion (V/Q) scan

QID: 33185

Option E (Ventilation-perfusion [V/Q] scan) is correct. This patient has a likely pulmonary embolism (PE). The diagnosis is suspected because of the finding of pleuritic chest pain (pain that changes with respiration), dyspnea, tachypnea, tachycardia, and new onset right bundle branch block. Patients can also be febrile with a PE. At present, ventilation-perfusion scan (V/Q scan) or spiral computed tomography (CT) scan are considered the first diagnostic steps in patients with an intermediate to high probability of having a pulmonary embolism. If the options had included anticoagulant therapy, that would have been the first step in the management of this patient, because of the reasonably high probability of this patient having a PE.

Option A (Echocardiography) is incorrect. Echocardiography is a rapid method of evaluating for the possibility of pulmonary embolism (PE) via right ventricular size or function. However, these findings are nonspecific and cannot be used to diagnose a PE. Its role in the management of patients with PE is largely confined after diagnosis to determine which patients can benefit from thrombolytic therapy.

Option B (Leg ultrasonography) is incorrect. Duplex ultrasonography of the lower extremities is useful if the ventilation/perfusion (V/Q) scan or spiral computed tomography (CT) scan findings are nondiagnostic for pulmonary embolism (PE).

Option C (Nebulized salbutamol) is incorrect. Nebulized salbutamol could possibly alleviate some of this patient's shortness of breath, but the issue causing the shortness of breath is delivery of blood to the lung, not oxygen into the alveoli. This would not be the first step in the management of this patient. The most appropriate first pharmacotherapy would be anticoagulation.

Option D (Pulmonary arteriography) is incorrect. Pulmonary arteriography is the gold standard for the diagnosis of pulmonary embolism (PE). However, there is a 4% risk of having a major complication during the procedure, including death, cardiac arrhythmias, bleeding, and cardiac perforation. As such, this is not the first diagnostic step.

Clinical features

Page 33: usmle questionnarie

Dyspnea is the most frequent symptom, and tachypnea is the most frequent sign in pulmonary embolism (PE). Dyspnea, syncope, hypotension, or cyanosis indicates a massive PE, while pleuritic pain, cough, or hemoptysis often suggests a small embolism located near the pleura. On examination, young or previously healthy individuals may simply appear anxious but otherwise seem well, even with a large PE.

Many drugs interact with warfarin to enhance or diminish its effect. Ask the patient to report any changes in treatment.

Investigations

Investigations supporting the diagnosis of PE include the following:

Electrocardiogram: tachycardia or the "typical" S1 Q3 T3 pattern. D-dimer, ESR, LDH, AST. ABGs: hypoxemia. CXR: usually normal but may show atelectasis or a small wedge shadow. V/Q scan: unmatched V/Q defects. PE protocol CT scan.

If these investigations are negative but the clinical suspicion remains high, a search for corroborative evidence of thromboembolism is instigated by measuring D-dimer and looking for the presence of deep vein thrombosis with ultrasound or venography. Patients who have a negative D-dimer and low probability of having PE are effectively ruled out with this test.

From Crash Course: Internal Medicine by Baliga

Figure 5.19 Ventilation:perfusion scan following pulmonary embolus. (Courtesy of Ivor Jones and the Nuclear Medicine staff, Derriford Hospital, Plymouth.)

From Crash Course: Respiratory System by Myers

A 75-year-old female presents to the emergency room with a 5-day history of intermittent nausea

and vomiting. She has also noticed increasing abdominal distension during this period. She has

been otherwise well and does not take any medications. Her previous medical and surgical

history is unremarkable. On examination, her vital signs are blood pressure (BP), 110/65 mm Hg;

pulse, 98 beats/minute; temperature, 38.4°C (101.1°F); and respirations, 14 breaths/minute.

Overall, she appears significantly dehydrated and is not jaundiced. Her abdomen is soft, but

distended and diffusely tender to deep palpation. Increased, high-pitched bowel sounds are

noted throughout the abdomen. Rectal examination is normal. Upright abdominal radiograph

demonstrates distended loops of small bowel with multiple air-fluid levels and air in the biliary

tree. What is the most likely diagnosis?

A. Adhesive small-bowel obstruction

Page 34: usmle questionnarie

B. Cholelithiasis

C. Gallstone ileus

D. Hepatic adenoma

E. Intussusception

F. Small intestine lymphoma

QID: 33074

Option C (Gallstone ileus) is correct. This patient has a gallstone ileus, where a large gallstone has travelled into the ileum via a biliary-enteric fistula causing obstruction. Gallstone ileus accounts for 25% of nonstrangulated small bowel obstruction in patients older than 65 years of age. Classically, the symptoms of gallstone ileus are subacute over a period of 3 to 10 days while the gallstone travels through the small bowel until becoming permanently lodged in the ileum. Abdominal radiograph is the first and best investigation for gallstone ileus and demonstrates the signs of small bowel obstruction combined with pneumobilia (air in the biliary tree).

Option A (Adhesive small-bowel obstruction) is incorrect. This patient has never had abdominal surgery and would therefore be unlikely to have an adhesion resulting in small-bowel obstruction.

Option B (Cholelithiasis) is incorrect. As gallstones and cholecystitis are likely the inciting event for creating the biliary-enteric fistula, this patient likely has cholelithiasis. However, the cholelithiasis is a likely incidental diagnosis that is not responsible for her present illness.

Option D (Hepatic adenoma) is incorrect. Hepatic adenomas are classically associated with oral contraceptive use, anabolic steroid use and glycogen storage diseases. They most frequently present with right upper quadrant pain in patients between 20 and 40 years old.

Option E (Intussusception) is incorrect. Intussusception is much more commonly found in the pediatric population as a cause of small bowel obstruction. Further, the finding of pneumobilia makes gallstone ileus more likely.

Option F (Small intestine lymphoma) is incorrect. Small bowel lymphomas are uncommon in the United States, but do have symptoms similar to the present case. They present with abdominal pain and bowel obstruction. In this case, the finding pneumobilia makes gallstone ileus more likely.

11. What is gallstone ileus?

Gallstone ileus is an unusual complication of cholelithiasis, most often occurring in elderly women. Gallstones, often greater than 2 cm in diameter, enter the intestines through a cholecystoduodenal fistula and migrate through the gastrointestinal tract. A mechanical obstruction occurs most frequently in the terminal ileum or at the ileocecal valve, where the

Page 35: usmle questionnarie

small intestine is the narrowest. Treatment is surgical.

From Hospital Medicine Secrets by Glasheen

A biotechnology company is seeking to develop a new test for the detection of diabetes using an

optical finger probe similar to a pulse oximeter. They screen 1000 random individuals using the

current diagnostic criteria of having two fasting blood glucose readings more than126 mg/dL.

They compare the results obtained with their new test to those of a standard blood test. Their

results are as follows:

Healthy DiabetesPositive optical test 50 90Negative optical test 850 10

What is the accuracy of the new optical test?

A. 10%

B. 64%

C. 90%

D. 94%

E. 99%

QID: 33417

Option D (94%) is correct. Accuracy is defined as the proportion of individuals who are correctly diagnosed by the screening test. Using the equation (true positive [TP] + true negative [TN])/1000, we get (850 + 90)/1000 = 94%.

Option A (10%) is incorrect. The prevalence of diabetes found in this random population of individuals can be calculated as the TPs plus the false negatives (FNs) divided by 1000 individuals. This calculation yields a 10% prevalence of diabetes.

Option B (64%) is incorrect. Positive predictive value is the proportion of people with a positive test who actually have the condition. Using the formula TP/(TP+ false positive [FP]), we get 90/(90 + 50) = 64%.

Option C (90%) is incorrect. Sensitivity is defined as the proportion of individuals who have the condition that are correctly identified by the screening test. Using the formula TP/(TP+ FN), we get 90/(90+10) = 90%.

Option E (99%) is incorrect. The negative predictive value is how likely a negative test is truly negative. Using the formula TN/(TN+ FN), we get 850/(850 + 10) = 99%.

Page 36: usmle questionnarie

Validity of Screening Tests

The validity of a test is defined as its ability to distinguish between who has a disease and who does not. Validity has two components: sensitivity and specificity. The sensitivity of the test is defined as the ability of the test to identify correctly those who have the disease. The specificity of the test is defined as the ability of the test to identify correctly those who do not have the disease.

Tests with Dichotomous Results (Positive or Negative)

Suppose we have a hypothetical population of 1,000 people, of whom 100 have a certain disease and 900 do not. A test is available that can yield either positive or negative results. We want to use this test to try to distinguish persons who have the disease from those who do not. The results obtained by applying the test to this population of 1,000 people are shown in Table 5-1.

From Epidemiology 3E by Gordis

Table 5-1. Concept of the Sensitivity and Specificity of Screening Examinations

Example: Assume a population of 1,000 people, of whom 100 have a disease and 900 do not have the disease

Screening Test to Identify the 100 People with the Disease

  True Characteristics in the Population  

Results of Screening Disease No Disease Total

Positive 80 100 180

Negative 20 800 820

Total 100 900 1,000

From Epidemiology 3E by Gordis

How good was the test? First, how good was the test in correctly identifying those who had the disease? Table 5-1 indicates that of the 100 people with the disease, 80 were correctly identified as "positive" by the test, and a positive identification was missed in 20. Thus, the sensitivity of the test, which is defined as the proportion of diseased people who were correctly identified as "positive" by the test, is 80/100 or 80%.

Second, how good was the test in correctly identifying those who did not have the disease? Looking again at Table 5-1, of the 900 people who did not have the disease, the test correctly identified 800 as "negative." The specificity of the test, which is defined as the proportion of nondiseased people who are correctly identified as negative by the test, is therefore 800/900 or 89%.

Note that to calculate the sensitivity and specificity of a test, we must know who "really" has the disease and who does not from another source than the test we are using. We are in fact comparing our test results with some "gold standard"-an external source of "truth" regarding the disease status of each individual in the population. Sometimes this truth may be the result of another test that has been in use, and sometimes it is the result of a more definitive, and often more invasive, test (e.g., cardiac catheterization or tissue biopsy). However, in real life, when we use a test to identify diseased and nondiseased persons in a population, we

Page 37: usmle questionnarie

clearly do not know who has the disease and who does not. (If this were already established, testing would be pointless.) But to quantitatively assess the sensitivity and specificity of a test, we must have another source of truth with which to compare the test results.

From Epidemiology 3E by Gordis

A 32-year-old gravida I, para 0 female presents to the labor and delivery ward following a large

discharge of clear fluid from her vagina 1 hour ago. Her pregnancy thus far as been

unremarkable except for positive third-trimester vaginal group B Streptococcus cultures. On route

to the hospital, she begins to have regular contractions and progresses to full cervical dilation

after 10 hours. Intrapartum penicillin is administered. A compound foot presentation is discovered

and immediate cesarean section is undertaken. A 3.0-kg (6.6 lb) male neonate is delivered with 1

and 5 minute APGAR scores of 9 and 9, respectively. At 1 hour of age, the infant is heard

grunting on expiration. He is tachypneic and has mild nasal flaring. He is tachycardic, but S1 and

S2 heart sounds are auscultated and there is no murmur. The abdomen is soft, without

organomegaly and there are no masses palpated. There are no petechiae or purpura and the

skin appears pale pink. A chest X-ray (CXR) reveals prominent perihilar streaking and fluid within

the horizontal fissure. During the physical examination, he appears to become cyanosed and

supplemental oxygen is commenced, resulting in a dramatic improvement in his symptoms. What

is the most likely diagnosis?

A. Neonatal sepsis

B. Persistent pulmonary hypertension of the newborn

C. Pneumothorax

D. Respiratory distress syndrome (RDS)

E. Transient tachypnea of the newborn

QID: 33092

Option E (Transient tachypnea of the newborn) is correct. Transient tachypnea of the newborn is the most common cause of respiratory distress in term infants. It is caused by a delay in the resorption of lung fluid and is more common following delivery by cesarean section. As a result, the chest X-ray (CXR) is the most characteristic finding in the question: perihilar streaking and fluid noted within the horizontal fissure.

Option A (Neonatal sepsis) is incorrect. The rapid improvement on administration of supplemental oxygen argues against neonatal sepsis as a cause of respiratory distress in this infant.

Option B (Persistent pulmonary hypertension of the newborn) is incorrect. Persistent

Page 38: usmle questionnarie

pulmonary hypertension results when there is a failure of the pulmonary vascular resistance to decrease at birth. This results in persistence of the fetal circulation and bypass of the lungs causing hypoxemia. This diagnosis is usually suggested when there is a history of perinatal asphyxiation. In the present case, the classic radiograph findings and rapid improvement on supplemental oxygen are more in keeping with transient tachypnea of the newborn.

Option C (Pneumothorax) is incorrect. Approximately 2% of newborns experience spontaneous, asymptomatic pneumothorax. There is no radiographic evidence of pneumothorax in this case.

Option D (Respiratory distress syndrome [RDS]) is incorrect. The classic finding of respiratory distress syndrome (RDS) is a uniform reticulonodular or “ground-glass” appearance with prominent air bronchograms. This syndrome occurs more frequently in premature infants.

7. In addition to RDS, what are other neonatal diffuse lung diseases? How might they be differentiated clinically?

Other neonatal diffuse lung diseases include transient tachypnea of the newborn (TTN), congestive heart failure (CHF), neonatal pneumonia, and meconium aspiration. Neonatal pneumonia may occur in the preterm or term infant. Meconium aspiration and TTN are generally diseases of term infants, and CHF and pneumonia may occur in either.

Figure 57-1 Frontal chest radiograph of a premature infant with diffuse ground-glass lung disease and low lung volumes, which is indicative of RDS. (Courtesy Richard Markowitz, Children's Hospital of Philadelphia.)

KEY POINTS: CAUSES OF NEONATAL DIFFUSE LUNG DISEASE1. Respiratory distress syndrome 2. Transient tachypnea of the newborn 3. Congestive heart failure 4. Pneumonia 5. Meconium aspiration

From Radiology Secrets 2E by Pretorius

A 10-year-old boy is brought to the emergency department by his parents following the

appearance of a rash 6 hours earlier. The parents report that the boy has been sick with a sore

throat and non-productive cough for the last three days, but did not seek treatment as he was

eating well and otherwise active. However this morning the boy began to complain of malaise,

general muscle aches, and a headache, and his parents measured an oral temperature of 39°C

(102.2°F). Several hours later the boy suddenly developed a rash, which is described as red

spots. The lesions appeared at the same time all over his body and are progressively worsening.

There is no history of sick contacts or recent travel. The boy has no other medical problems and

no known allergies. Vital signs are significant for hypotension, tachycardia, and a temperature of

40°C (104°F). The patient appears anxious and restless but is alert. Physical examination is

significant for a diffuse, erythematous papular rash, shown. The rash is present on the trunk and

Page 39: usmle questionnarie

extremities, but not the face, palms, and soles. There is no blanching of the lesions when

pressure is applied with a glass slide. No nuchal rigidity is appreciated, and Kernig’s and

Brudzinski’s signs are negative. Blood tests shown. What is the most appropriate treatment at

this time?

WBC (white blood cell count) Hb (hemoglobin) Hct (hematocrit) Platelets

14 15 45 100

Na K Cl HCO3 BUN (blood urea nitrogen) Creatinine Glucose

142 4.5 104 22 14 0.6 100

PT (prothrombin time) aPTT (activated partial thromboplastin time)

17 40

D-dimer Fibrin degradation products

Page 40: usmle questionnarie

15 300

A. Doxycycline

B. Penicillin G

C. Plasma exchange

D. Prednisone

E. Supportive therapy

QID: 47967

Option B (Penicillin G) is correct. This is the 1st line treatment for meningococcemia, which is a systemic infection by the bacteria Neisseria meningitides. It is contracted by inhalation of airborne droplets of infected nasopharyngeal secretions, which then spread hematogenously and can result in meningitis and fulminant meningococcemia. Patients usually present with an antecedent upper respiratory infection, which is followed by high fever, chills, myalgias, and other constitutional symptoms. Changes in mental status may also occur. The hallmark rash reflects systemic endothelial damage and diffuse intravascular coagulation (DIC), and is characterized by the sudden appearance of widespread purpuric lesions with necrotic centers that do not blanche under pressure. This patient’s history, appearance of the rash, and lab values are strongly consistent with this diagnosis and empirical antibiotic treatment should be immediately initiated.

Option A (Doxycycline) is incorrect. This is the treatment of choice for Rocky Mountain spotted fever, which is a tick-borne disease caused by the organism Rickettsia rickettsii. Patients classically present with a history of tick exposure, fever, and a petechial rash that begins on the palms and soles and spreads centripetally. This is an unlikely diagnosis in this patient given the lack of history of tick exposure and absence of lesions on the palms and soles. Furthermore, the lesions illustrated are purpuric, which coupled with an antecedent upper respiratory infection and evidence of consumptive coagulopathy is strongly suggestive of fulminant meningococcemia.

Option C (Plasma exchange) is incorrect. This is the treatment for thrombotic thrombocytopenic purpura, which is a hematological disorder resulting from the deposition of abnormal von Willebrand factor multimers in the vasculature, with concomitant platelet consumption and microangiopathic hemolytic anemia. It is most often idiopathic. The classic diagnostic pentad includes mental status changes, thrombocytopenia, microangiopathic hemolytic anemia, neurological symptoms such as change in mental status, and evidence of nephritis or overt renal failure. This patient’s history, appearance of the rash, leukocytosis, and normal hematocrit and creatinine are more consistent with fulminant meningococcemia.

Option D (Prednisone) is incorrect. This is the primary treatment for idiopathic thrombocytopenic purpura, which is an acute, self-limiting hematological disorder caused by antibody-mediated clearance of platelets. It is often preceded by a viral infection or recent

Page 41: usmle questionnarie

immunization, and is a diagnosis of exclusion. This patient’s history, symptom complex, leukocytosis, and evidence of DIC are not compatible with this diagnosis and are strongly suggestive of an infectious process.

Option E (Supportive therapy) is incorrect. This is the initial treatment of Henoch-Schönlein purpura, which is a systemic immunoglobulin A (IgA)-mediated vasculitis common in children. It is idiopathic, but often follows an upper respiratory infection. It typically presents as “palpable purpura” affecting the buttocks and lower extremities, and is accompanied by fever, arthralgias, abdominal colic, and nephritis. Purpura is a manifestation of endothelial damage secondary to immune complex deposition and not abnormal platelet number or function. This patient’s distribution of the rash, symptoms, leukocytosis, thrombocytopenia, normal creatinine, and evidence of a consumptive coagulopathy are strongly suggestive of meningococcemia and requires definitive intervention.

Neisseria meningitidis

1. Classification a. Serogroups based on the polysaccharide capsule; most common = A, B, C,

Y, and W135 b. Serotypes based on the outer membrane protein c. Immunotypes based on LPS

2. Pathogenesis a. Neisseria meningitidis enters the respiratory tract, invades mucous

membranes, and spreads via the bloodstream. b. Antiphagocytic capsule is important for virulence. c. Released endotoxin induces fever and increases vascular permeability,

potentially leading to shock and petechiae (capillary leakage in skin). 3. Meningococcal diseases

o Meningococcal infection is most common in children younger than 5 years of age and those with deficiency of terminal complement components (C5-C9).

b. Meningitis: acute onset of fever, headache, and stiff neck c. Acute meningococcemia: septicemia with or without meningitis

Characterized by fever, shock, and generalized hemorrhage ranging from petechiae to purpura

Can be rapidly fatal (mortality rate of 25% or higher) if not treated promptly

d. Waterhouse-Friderichsen syndrome: complication of meningococcemia Marked by overwhelming disseminated intravascular coagulation

and bilateral hemorrhagic adrenal necrosis with septic shock, acute hypotension, tachycardia, and petechiae

e. Chronic meningococcemia: milder disease characterized by persistent (for weeks) bacteremia

Low-grade fever, arthritis, and petechial skin lesions f. Mild febrile disease with pharyngitis, pneumonia, arthritis, or urethritis

Transmission of N. meningitidis a. Inhalation of aerosol droplets (often from asymptomatic nasopharyngeal

carriers) b. Person-to-person spread from infected persons

Prevention a. Breast-feeding infants for the first 6 months of life b. Active immunization of children older than 2 years of age with a polyvalent

Page 42: usmle questionnarie

conjugate, anticapsular vaccine (not effective against serogroup B) c. Postexposure prophylaxis with rifampin, quinolones, or sulfonamides (only

if the organism is proven susceptible) Treatment

a. Penicillin is the drug of choice. Alternatives include broad-spectrum cephalosporins (ceftriaxone), chloramphenicol, and sulfonamides (if susceptible).

b. Antibiotics must be secreted from mucosa to eliminate carrier state.

From Rapid Review Microbiology & Immunology 2E by Rosenthal & Tan

A 46-year-old man has left lower abdominal pain, and a CT scan demonstrates sigmoid colon

thickening and mesenteric “stranding.” After three days of IV antibiotics and inpatient

hospitalization for this episode of presumed acute diverticulitis, he is sent home on oral

antibiotics. He develops profuse watery diarrhea and crampy abdominal pain over the

subsequent two to three days. Which of the following is the most likely diagnosis?

A. Antibiotic-associated colitis

B. Ischemic colitis

C. Missed appendicitis

D. Partial colonic obstruction

E. Recurrent diverticulitis

QID: 27701

Option A (Antibiotic-associated colitis) is correct. Clostridium difficile colitis is common after antibiotic therapy. It can be treated by oral antibiotics in most cases.

Option B (Ischemic colitis) is incorrect. Ischemic colitis is related to low-flow–induced colonic ischemia. The history and presentation are not consistent with this diagnosis.

Option C (Missed appendicitis) is incorrect. The initial CT findings and location of the pain are consistent with the diagnosis of acute diverticulitis. The watery diarrhea and abdominal symptoms do not call the original diagnosis into doubt but rather suggest evolution of antibiotic-associated enterocolitis.

Option D (Partial colonic obstruction) is incorrect. Large bowel obstruction may occur chronically if a stricture develops as a result of repeated attacks of inflammation or in the setting of untreated acute inflammation. However, in this case, the resolution of the toxic symptoms and the evolution of the watery diarrhea during antibiotic therapy make antibiotic-associated enterocolitis more likely.

Page 43: usmle questionnarie

Option E (Recurrent diverticulitis) is incorrect. This is not likely if fevers and focal tenderness have not recurred.

C. difficile is part of the normal intestinal flora in a small number of healthy people and hospitalized patients. The disease develops in people taking antibiotics, because the drugs alter the normal enteric flora, either permitting the overgrowth of these relatively resistant organisms or making the patient more susceptible to the exogenous acquisition of C. difficile. The disease occurs if the organisms proliferate in the colon and produce their toxins.

The diagnosis of C. difficile infection is confirmed by demonstration of the enterotoxin or cytotoxin in a stool specimen from a patient with compatible clinical symptoms. Isolation of the organism in stool culture documents colonization but not disease. The enterotoxin and cytotoxin can be detected with a number of commercial immunoassays. These assays vary tremendously in sensitivity and specificity, so care must be used in selecting the appropriate test, and a negative test result does not exclude the diagnosis. The cytotoxin can also be detected by an in vivo cytotoxicity assay using tissue culture cells and specific neutralizing antibodies for the cytotoxin; however, this assay is technically cumbersome and requires 1 to 2 days before results are available. Most laboratories have replaced the cytotoxicity assay with immunoassay methods.

Discontinuation of the implicated antibiotic (e.g., ampicillin, clindamycin) is generally sufficient to alleviate mild disease. However, specific therapy with metronidazole or vancomycin is necessary for the management of severe diarrhea or colitis. Relapses may occur in as many as 20% to 30% of patients after the completion of therapy, because only the vegetative forms of C. difficile are killed by the antibiotics; the spores are resistant. A second course of treatment with the same antibiotic is frequently successful. It is difficult to prevent the disease, because the organism commonly exists in hospitals, particularly in areas adjacent to infected patients (e.g., beds, bathrooms). The spores of C. difficile are difficult to eliminate unless thorough housekeeping measures are used. Thus the organism can contaminate an environment for many months and can be a major source of nosocomial outbreaks of C. difficile disease.

Taken from Medical Microbiology 5E by Murray et al

BOX 40-6. Summary of Clostridium difficilePhysiology and Structure

Gram-positive, spore-forming rod Strict anaerobe (vegetative cells are extremely oxygen sensitive)

Virulence

Refer to Table 40-4

Epidemiology

The organism is ubiquitous Colonizes the intestines of a small proportion of healthy

individuals (<5%) Exposure to antibiotics is associated with overgrowth of C.

difficile and subsequent disease (endogenous infection) Spores can be detected in hospital rooms of infected patients

Page 44: usmle questionnarie

(particularly around beds and in the bathrooms); these can be an exogenous source of infection

Diseases

Refer to Box 40-3

Diagnosis

C. difficile disease is confirmed by detecting the cytotoxin or enterotoxin in the patient's feces

Treatment, Prevention, and Control

The implicated antibiotic should be discontinued Treatment with metronidazole or vancomycin should be used in

severe disease Relapse is common because antibiotics do not kill spores; a

second course of therapy with the same antibiotic is usually successful

The hospital room should be carefully cleaned after the infected patient is discharged

Taken from Medical Microbiology 5E by Murray et al

C. difficile produces two toxins (Table 40-5), an enterotoxin (toxin A) and a cytotoxin (toxin B). The enterotoxin is chemotactic for neutrophils, stimulating the infiltration of polymorphonuclear neutrophils into the ileum with release of cytokines. Toxin A also produces a cytopathic effect, resulting in disruption of the tight cell-cell junction, increased permeability of the intestinal wall, and subsequent diarrhea. The cytotoxin causes actin to depolymerize, with the resultant destruction of the cellular cytoskeleton both in vivo and in vitro. Although both toxins appear to interact synergistically in the pathogenesis of disease, enterotoxin A-negative isolates can still produce disease. Additionally, production of one or both toxins does not appear to be sufficient alone for disease (e.g., carriage of C. difficile and high levels of toxins are common in young children while disease is rare). Bacterial "surface layer proteins" (SLPs) are important for the binding of C. difficile to the intestinal epithelium, leading to localized production of toxins and subsequent tissue damage. Other C. difficile virulence factors are summarized in Table 40-5.

Taken from Medical Microbiology 5E by Murray et al

A 51-year-old man is brought into the emergency room after three episodes of vomiting bright red

blood. He has a known history of cirrhosis secondary to alcoholic liver disease. On arrival, his

vital signs are as follows: blood pressure, 90/60 mm Hg; pulse, 115 beats/minute; temperature

37.6°C (99.6°F); and respirations, 18 breaths/minute. His abdomen is dull to percussion

throughout. A nasogastric tube is inserted, and bright red blood is found when stomach contents

are aspirated. What is the most appropriate next step in the management of this patient?

Page 45: usmle questionnarie

A. Balloon tamponade

B. Endoscopic band ligation

C. Intravenous normal saline

D. Octreotide

E. Propranolol

QID: 33275

Option C (Intravenous [IV] normal saline) is correct. This patient has an upper gastrointestinal bleed, most likely the result of bleeding esophageal varices. However, the exact cause is unimportant in this case, because the patient is hemodynamically unstable. He is hypotensive and tachycardic and thus requires hemodynamic resuscitation. The option of IV normal saline meets this requirement.

Option A (Balloon tamponade) is incorrect. Balloon tamponade is reserved for patients who continue to have bleeding esophageal varices despite endoscopic band ligation.

Option B (Endoscopic band ligation) is incorrect. If medical therapy with octreotide fails, endoscopic band ligation is the next step in the management of bleeding esophageal varices.

Option D (Octreotide) is incorrect. Were this patient hemodynamically stable, the use of octreotide is considered the first step in the management of bleeding esophageal varices.

Option E (Propranolol) is incorrect. Propranolol and other nonselective beta blockers are used as prophylaxis against bleeding esophageal varices. There is no role acutely, especially not in hypotensive patients.

APPROACH TO THE PATIENT WITH ACUTE GASTROINTESTINAL BLEEDING (Fig. 32-2)

Assessment of Vital Signs/Resuscitation

The first step in the evaluation and therapy for the patient with acute GI hemorrhage is to determine the severity of blood loss. Vital signs should be recorded immediately. If the systolic blood pressure drops more than 10 mm Hg and/or the pulse increases more than 10 beats per minute as the patient changes positions from supine to standing, it is likely the patient has lost at least 800 mL (15%) of circulating blood volume. Hypotension, tachycardia, tachypnea, and mental status changes in the setting of acute GI hemorrhage suggest at least a 1500-mL (30%) loss of circulating blood volume.

The goal of resuscitation is to restore the normal circulatory volume. Initially, at least two large-bore intravenous catheters are used to administer isotonic solutions (e.g., lactated Ringer's solution or 0.9% NaCl), and blood products if indicated. If the patient is in shock, a central venous access should be established. The amount of blood products to be

Page 46: usmle questionnarie

transfused must be individualized. Transfusions of packed red blood cells are provided to prevent complications (e.g., angina, congestive heart failure, stroke) of acute blood loss. Therefore, the need for blood transfusion depends on multiple factors, including the patient's age, overall health, and response of vital signs to initial resuscitation. In view of the risks of blood transfusion, it is not appropriate simply to transfuse until an arbitrary hematocrit is achieved. If coagulation studies are abnormal, as commonly observed in cirrhotic patients, fresh-frozen plasma and/or platelets may be required to control ongoing hemorrhage. Opinions differ regarding the use of nasogastric lavage in preparation for endoscopic examination, although aspiration of gastric blood may be particularly important in patients with liver cirrhosis, because blood in the GI tract may precipitate hepatic encephalopathy.

From Cecil Essentials of Medicine 6E by Andreoli et al

Table 32-3. Common Sources of Acute Gastrointestinal Hemorrhage

Source Associated Clinical Features TreatmentsUpper Gastrointestinal Tract

Esophagitis Heartburn, dysphagia, odynophagia Medication*Antireflux surgery or procedures

Esophageal cancer Progressive dysphagia, weight loss Chemoradiotherapy, surgeryPalliative endoscopy procedures

Gastritis/gastric ulcer Aspirin/NSAID use Withdraw NSAIDs

Duodenitis/duodenal ulcer Abdominal pain/dyspepsia Medication†

Helicobacter pylori infection Endoscopic therapy for acute bleeding

Gastric cancer Early satiety, weight loss, abdominal pain Surgery, chemotherapy

Esophagogastric varices History of CLD Variceal banding, sclerotherapy

Stigmata of CLD on examination Vasopressin, octreotideTIPS or decompressive surgery

Mallory-Weiss tear History of retching before hematemesis Supportive (usually self-limited)Endoscopic therapy

Lower Gastrointestinal Tract

Infection History of exposure, diarrhea, fever Supportive/antibiotics

Inflammatory bowel diseases

History of colitis, diarrhea, abdominal pain, fever

Steroids/5-ASA/immunotherapySurgery if no response to medication

Diverticula Painless hematochezia SupportiveSurgery for recurrent disease

Angiodysplasia Painless hematochezia Endoscopic therapy

Often in ascending colon Supportive

Commonly involves stomach and small bowel as well

Surgery for localized disease

Colon cancer Change in bowel habit, anemia, weight loss Surgery

Colon polyp Usually asymptomatic Endoscopic or surgical removal

Ischemic colitis Typically elderly patientsHistory of vascular diseaseMay present with abdominal pain

Supportive (self-limited)

Meckel's diverticulum Painless hematochezia in young patientLocated at distal ileum

Surgery

Hemorrhoids Rectal bleeding associated with bowel movement

Supportive

From Cecil Essentials of Medicine 6E by Andreoli et al

Page 47: usmle questionnarie

Figure 32-2 Approach to the patient with acute gastrointestinal bleeding. EGD = esophagogastroduodenoscopy.

A 73-year-old woman is brought to the emergency room because this morning she was found by

her daughter to be acutely confused. She has otherwise been in excellent health and does not

take any regular medication. The daughter reports that her mother's mental status was normal

yesterday. On arrival in the emergency room, she is disorientated to person, place, and time.

She is diaphoretic and tremulous. Her blood pressure is 125/85 mm Hg; pulse, 88 beats/minute;

temperature, 37.4°C (99.3°F); and respirations, 14 breaths/minute. Occasional ectopic heart

beats are detected during auscultation of the heart. There is bilaterally symmetrical 4/5 muscle

strength in the upper and lower extremities. Laboratory evaluation following an overnight fast

demonstrates the following:

SerumSodium 136 mEq/LChloride 97 mEq/LPotassium 4.1 mEq/LBicarbonate 22 mEq/LMagnesium 1.6 mEq/LCalcium 8.4 mg/dLUrea nitrogen (BUN) 11 mg/dLCreatinine 0.6 mg/dLGlucose 40 mg/dLInsulin 15 micro-U/mL (Normal <9 micro-U/L)Pro-insulin 12 micro-U/mL (Normal <20% total insulin)C-peptide 3.01 ng/mL (Normal 0.78 – 1.89 ng/L)

What is the most likely cause of this patient's findings?

A. Alpha-cell pancreatic endocrine neoplasm

B. Beta-cell pancreatic endocrine neoplasm

C. Delta-cell pancreatic endocrine neoplasm

D. Exogenous insulin

E. Sulfonylurea use

QID: 33220

Page 48: usmle questionnarie

Option B (Beta-cell pancreatic endocrine neoplasm) is correct. This patient has an insulinoma, the most common pancreatic endocrine neoplasm. The diagnosis is based on symptoms of hypoglycemia (confusion, palpitations, diaphoresis) and laboratory evaluation showing blood glucose less than 50 mg/dL, elevated insulin, pro-insulin and C-peptide.

Option A (Alpha-cell pancreatic endocrine neoplasm) is incorrect. Alpha-cell tumors are glucagonomas and result more commonly in hyperglycemia. There is a characteristic rash associated with alpha-cell tumors known as necrolytic migratory erythema. There is often an associated anemia.

Option C (Delta-cell pancreatic endocrine neoplasm) is incorrect. These neoplasms are also known as somatostatinomas and typically cause cholelithiasis, steatorrhea, and hypochlorhydria. There would be elevated somatostatins.

Option D (Exogenous insulin) is incorrect. Exogenous insulin suggests that insulin is being injected. Such insulin would not have the C-peptide in it, and as a result, C-peptide would not be elevated.

Option E (Sulfonylurea use) is incorrect. Sulfonylureas result in increased insulin output, but not increased insulin production. Consequently, although there can be elevated insulin and C-peptide, pro-insulin should be normal if the cause were sulfonylurea administration.

7. What are the symptoms of hypoglycemia?

Hypoglycemic symptoms are classified according to the their type and their timing in relation to meals. Symptoms, such as confusion, slurred speech, blurred vision, seizures, and coma, result from inadequate delivery of glucose to the brain (neuroglycopenia). Symptoms, such as tremors, sweating, palpitations, and nausea, result from a counter-regulatory discharge of catecholamines (adrenergic). When symptoms occur within 5 hours of the previous meal, they are considered to be "postprandial"; if they occur more than 5 hours after a meal they are considered to be "fasting." Insulinomas most commonly cause fasting neuroglycopenic symptoms, although postprandial and adrenergic symptoms may also occur.

From Endocrine Secrets 4E by McDermott

8. How is the diagnosis of an insulinoma made?

The diagnosis requires documentation of symptomatic hypoglycemia and endogenous hyperinsulinemia. Hyperinsulinemic hypoglycemia is defined as hypoglycemia with a insulin:glucose ratio of > 0.33. Some patients present to the provider during a symptomatic episode, but more commonly the physician must attempt to provoke hypoglycemia when the diagnosis is suspected. This often requires a prolonged fast (up to 48 hours) with blood sampling for glucose, insulin and C-peptide levels every 6 hours and during any symptoms that occur. Since proinsulin is normally cleaved into insulin and C-peptide within the pancreas, both C-peptide and proinsulin are markers of endogenous insulin secretion. A drug screen for sulfonylurea and meglitinide use is also recommended.

9. How can insulinomas be distinguished from other causes of hyperinsulinemic hypoglycemia?

Hyperinsulinemic hypoglycemia can be due to insulinomas, surreptitious insulin administration, and medication use. Table 55-1 illustrates how these entities can be distinguished.

Page 49: usmle questionnarie

From Endocrine Secrets 4E by McDermott

Table 55-1. INSULINOMAS VERSUS OTHER CAUSES OF HYPERINSULINEMIC HYPOGLYCEMIA

Test InsulinomaSurreptitiousInsulin Use

Sulfonylurea orMeglitinide Use

Insulin ↑ ↑ ↑

C-peptide ↑ ↓ ↑

Proinsulin ↑ ↓ Nl

Drug screen Neg Neg Pos

From Endocrine Secrets 4E by McDermott

A 19-year-old woman is brought to the physician by her parents because they have noticed that

she is increasingly losing weight. She states that there is nothing wrong with her behavior and

that she is purposefully trying to lose weight simply because she is overweight. Her parents state

that her sole hobby is to photograph food and that she has forsaken all other activities to devote

herself to food photography. Further questioning reveals that she is intensely fearful of being

overweight. A review of systems finds that she has not had menses in the last 4 months despite

no sexual activity. Physical examination reveals a body mass index (BMI) of 15 kg/m2 (15 lb/in2),

lanugo, and dry skin. What is the most likely diagnosis?

A. Anorexia nervosa

B. Binge-eating disorder

C. Body dysmorphic disorder

D. Bulimia nervosa

E. Schizophrenia

QID: 33512

Option A (Anorexia nervosa) is correct. The diagnosis of anorexia requires four elements: a refusal to maintain appropriate body weight, an intense fear of gaining weight, undue influence of body shape/weight on self-image, and amenorrhea (in postmenarchal females). Some anorexics can purge, but the behavior is not primary and not as frequent as in bulimia. Also, note for the diagnosis that the patient must be below 85% of minimally recommended weight. In this case, all four criteria are met.

Option B (Binge-eating disorder) is incorrect. Binge-eating disorder occurs in overweight individuals where there is binge eating and no compensatory behavior.

Page 50: usmle questionnarie

Option C (Body dysmorphic disorder) is incorrect. Binge-eating disorder describes binge eating without compensatory behavior. Consequently, these individuals are usually obese.

Option D (Bulimia nervosa) is incorrect. The diagnosis of bulimia requires that there be episodes of binge eating followed by inappropriate compensatory behavior. The patient must be unduly influenced by body size/weight, and the behavior must exist twice a week for 3 months. Note that the compensatory behavior does not need to be purging in nature. Excessive exercise, abuse of laxatives, enemas, and diuretics are also considered inappropriate compensatory behaviors.

Option E (Schizophrenia) is incorrect. In schizophrenia, there must be hallucinations, delusions, or significant negative symptoms for a period longer than 6 months. Although this person feels that they are overweight when they have a BMI of 15 kg/m2 (15 lb/in2), this is not considered a delusion.

Table 17-1. Comparison of Anorexia Nervosa and Bulimia Nervosa

Disorder FeaturesComorbid Conditions Complications*

Anorexia nervosa

Maintenance of weight <85% normal for age and heightDistorted perceptions of own body sizeIntense fear of gaining weight or becoming fat; self-esteem largely determined by weightAmenorrhea: absence of at least three consecutive menstrual cyclesDenial of serious health effects of very low body weightRestricting type: obsessive-compulsive behavior, social withdrawal, restrained emotional expression, strong need for control

Depressive disordersObsessive-compulsive disorderPersonality disorders

HypotensionBradycardiaHypothermiaLeukopenia, anemiaOsteoporosisDry skin, lanugo (fine, downy hair)Cardiac arrhythmias

Bulimia nervosa

Repeated binge eatingUse of inappropriate methods to prevent weight gainSense of lack of control during binge-eating episodesTypically normal weightSelf-esteem largely determined by weightRestricted intake between bingesHiding binge eating and purging from othersImpulsive behaviors (e.g., stealing, promiscuity, overspending)

Depressive disordersAnxiety disordersSubstance use disordersPersonality disorders (frequently borderline personality disorder)

Loss of dental enamelFluid and electrolyte disturbancesCardiac arrhythmiasSwollen parotid glandsCalluses on back of hand from self-induced vomitingEsophageal tears or rupturesDehydration

From Rapid Review Behavioral Science 2E by Stevens et alA 55-year-old female is brought to the emergency room with a sudden onset of severe chest,

back, and abdominal pain that began 20 minutes ago. The pain is described as being very sharp

with a tearing-like character. Her previous medical history is notable for hypertension treated with

hydrochlorothiazide. She does not smoke tobacco or drink alcohol.

On examination, her blood pressure is 145/95 mm Hg in the right arm and 119/75 mm Hg in the

left arm; pulse, 105 beats/minute; temperature, 37.3°C (99.1°F); and respirations, 17

breaths/minute. A chest X-ray (CXR) demonstrates a widened mediastinum, and

electrocardiography (ECG) reveals nonspecific ST and T wave changes. What is the most

appropriate next step in the management of this patient?

Page 51: usmle questionnarie

A. Aortography

B. Emergent surgery

C. Intravenous labetalol

D. Thoracic spiral computed tomography (CT)

E. Transesophageal echocardiography (TEE)

QID: 33208

Option C (Intravenous labetalol) is correct. Ripping chest pain that radiates into the back and stomach, combined with a pulse deficit, previous history of hypertension and widened mediastinum on chest X-ray (CXR) should be considered an aortic dissection until proven otherwise. As soon as aortic dissection is suspected, immediate intravenous (IV) beta-blocker therapy should be started to reduce the pulse and blood pressure of the patient in an attempt to limit the severity of the dissection.

Option A (Aortography) is incorrect. Aortography is rarely performed, because it is invasive and requires contrast media into the aorta.

Option B (Emergent surgery) is incorrect. Emergent surgery is indicated in cases of aortic dissection where the proximal aorta is involved. The exact location is determined using computed tomography (CT) scanning and therefore, surgery would not be the next step in the management of this patient.

Option D (Thoracic spiral computed tomography [CT]) is incorrect. Spiral CT scanning is excellent for making the diagnosis of aortic dissection. However, the patient can die in the CT scanner without IV beta-blocker therapy.

Option E (Transesophageal echocardiography [TEE]) is incorrect. TEE is very useful in the diagnosis of aortic dissection and is the preferred imaging modality in very unstable patients. This patient is relatively stable, however and requires beta-blocker therapy.

Chest x-ray may show widened mediastinum (62%) and displacement of aortic intimal calcium.

Transesophageal echocardiography (TEE) is study of choice in unstable patients, but operator dependent.

MRI has the highest sensitivity and specificity but limited availability; not suitable for unstable patients; contraindicated with pacemakers, metal devices.

Helical CT is least operator dependent, involves intravenous contrast. TEE, MRI, helical CT are imaging modalities of choice. Sensitivities (98% to 100%)

and specificities (95% to 98%) nearly equal in skilled hands. Test of choice depends on clinical circumstances and availability.

With medium or high pretest probability, a second diagnostic test should be done if the first is negative.

Transthoracic echocardiography has poor sensitivity

Page 52: usmle questionnarie

Aortography rarely done now.

TREATMENT

ACUTE GENERAL Rx

Admit to ICU for monitoring. Propanolol 1 mg every 3-5 min or metoprolol 5 mg IV every 5 min, followed by

nitroprusside 0.3-10 mg/kg/min, with target SBP 100-120. IV beta-blocker is cornerstone of treatment. IV Labetalol can be used instead; 20 mg IV, then 40-80 mg every 10 min. IV calcium channel blockers with negative inotropy may be used. Multiple medications may be needed. Proximal dissections require emergent surgery to prevent rupture or pericardial

effusion. Distal dissections are treated medically unless distal organ involvement or impending

rupture occurs. Evolving role for endovascular stent placement as less invasive treatment for high-

risk surgical patients.

From Ferri's Clinical Advisor 2008 by Ferri

A 34-year-old gravida IV para III female at 37 weeks of gestation is brought to the emergency

room following the sudden onset of abdominal pain and vaginal bleeding 2 hours ago. The pain

has been constant from onset. She has also experienced frequent, strong contractions. She has

been otherwise well, abstinent from intercourse for the past 2 months, and her previous medical

and obstetric history are unremarkable. Her vital signs are blood pressure (BP), 100/65 mm Hg;

pulse, 110 beats/minute; temperature, 37.7°C (99.8°F); and respirations, 14 breaths/minute. The

uterus is painful and rigid. Speculum examination demonstrates mild vaginal bleeding. Fetal

heart rate tracing is not reassuring. What is the most likely diagnosis?

A. Abruptio placenta

B. Placenta previa

C. Spontaneous abortion

D. Uterine rupture

E. Vasa previa

QID: 33070

Option A (Abruptio placenta) is correct. This patient is demonstrating the classic signs of placental abruption: painful, abrupt vaginal bleeding associated with uterine contractions and

Page 53: usmle questionnarie

nonreassuring fetal heart tracing. The pain is constant, and the uterus is firm because of tetanic contractions. Vaginal bleeding can be highly variable and does not correlate well with the severity of abruption.

Option B (Placenta previa) is incorrect. Placenta previa is classically bright red painless bleeding that is not as rapid in onset as abruption. The uterus is usually soft.

Option C (Spontaneous abortion) is incorrect. Spontaneous abortion is defined as a pregnancy that has ended spontaneously before 20 to 22 weeks. At 37 weeks, this fetus is viable.

Option D (Uterine rupture) is incorrect. Uterine rupture is painful third trimester bleeding, but is very rare. Unlike abruptio placenta, it is associated with constant heavy vaginal bleeding. A classic presentation on the examination is a fetus that is at a +2 station that suddenly retracts into the -1 position.

Option E (Vasa previa) is incorrect. Vasa previa has a classic triad of membrane rupture followed by vaginal bleeding and then fetal bradycardia. Vasa previa is bleeding from fetal vessels and is diagnosed using the Apt test.

ABRUPTIO PLACENTAE

Abruptio placentae, or premature separation of the normally implanted placenta, complicates 0.5% to 1.5% of all pregnancies (1 in 120 births). Abruption severe enough to result in fetal death occurs in 1 per 500 deliveries.

From Essentials of Obstetrics & Gynecology 4E by Hacker et al

DIAGNOSIS AND MANAGEMENT

Clinically, the diagnosis of a placental abruption is entertained if a patient presents with painful vaginal bleeding in association with uterine tenderness, hyperactivity, and increased tone. The signs and symptoms of placental abruption are, however, variable. The most common finding is vaginal bleeding, seen in 80% of cases. Abdominal pain and uterine tenderness are seen in 66% of cases, fetal distress in 60%, uterine hyperactivity and increased uterine tone in 34%, and fetal demise in 15%.

Box 11-2. Risk factors for abruptio placentae Maternal hypertension Placental abruption in a prior pregnancy Trauma Polyhydramnios with rapid decompression Premature rupture of membranes Short umbilical cord Tobacco use Folate deficiency

The diagnosis of placental abruption is primarily a clinical one. Ultrasonography may detect only 2% of abruptions. Because placental abruption may coexist with a placenta previa, the reason for doing an initial ultrasonic examination is to exclude the latter diagnosis.

Page 54: usmle questionnarie

Management of the patient with an abruption includes careful maternal hemodynamic monitoring, fetal monitoring, serial evaluation of the hematocrit and coagulation profile, and delivery. Intensive monitoring of both the mother and the fetus is essential because rapid deterioration of either one's condition can occur. Blood products for replacement should always be available, and a large-bore (16- to 18-gauge) intravenous line must be secured. Red blood cells should be given liberally if indicated.

From Essentials of Obstetrics & Gynecology 4E by Hacker et al

When adenocarcinoma of the pancreas arises in older adults, molecular analysis of the

carcinoma cells reveals multiple genetic alterations from normal pancreatic ductal cells. A point

mutation involving which of the following genes, which leads to formation of a protein product that

is constitutively active, is most likely to be present in these carcinoma cells?

A. APC

B. HER2

C. K-RAS

D. p53 (TP53)

E. SPINK1

QID: 95445

Option C (K-RAS) is correct. RAS mutations are common in epithelial cancers, particularly adenocarcinomas, and are the most common gene mutations in pancreatic adenocarcinomas. The abnormally activated RAS protein drives intracellular signal transduction pathways that promote production of transcription factors driving cellular proliferation.

Option A (APC) is incorrect. APC gene mutations are most likely to be associated with adenocarcinomas of the colon.

Option B (HER2) is incorrect. HER2 mutations are seen in breast cancers.

Option D (p53 (TP53)) is incorrect.Although p53 (TP53) mutations are common in many forms of carcinoma, the mechanism is that of a tumor suppressor gone awry. p53 protein acts in cell cycle checkpoint pathways and in control of cellular apoptosis.

Option E (SPINK1) is incorrect. The SPINK1 and PRSS1 gene mutations are associated with hereditary pancreatitis.

The symptomatic course of pancreatic carcinoma is typically brief and progressive. Despite the tendency of lesions of the head of the pancreas to obstruct the biliary system, fewer than

Page 55: usmle questionnarie

20% of pancreatic cancers overall are resectable at the time of diagnosis. There has long been a search for biochemical tests that could be useful in the early detection of pancreatic cancer. The K-RAS oncogene is mutated in 90% of pancreatic cancers; however, the utility of screening tests for K-RAS mutations remains unproven. Serum levels of many enzymes and antigens (e.g., carcinoembryonic antigen and CA19-9 antigen) have been found to be elevated, but these markers are not specific nor are they sensitive enough to be used as screening tests. Several imaging techniques, such as endoscopic ultrasonography and CT, have proved of great value in diagnosis and the performance of percutaneous needle biopsy. Both of these techniques, while useful in establishing a diagnosis, are not useful as screening tests.

From Robbins & Cotran Pathologic Basis of Disease 7E by Kumar et al

A 44-year-old woman has experienced pain in her neck over the past 2 weeks. On physical

examination her thyroid is enlarged, firm, and tender to palpation. Her serum thyroxine is

increased. A fine needle aspiration biopsy shows multinucleated giant cells along with acute and

chronic inflammatory cells and destruction of thyroid follicles. At follow-up 2 months later she is

euthyroid. Which of the following is the most likely risk factor for development of her disease?

A. HLA-DR3 allele

B. Idiosyncratic drug reaction

C. Iodine deficiency

D. Prior radiation exposure

E. RET gene mutation

F. Viral pneumonia

QID: 95548

Option F (Viral pneumonia) is correct. She has a subacute granulomatous thyroiditis, known as DeQuervain disease. This form of thyroiditis is often preceded by a viral respiratory infection. It is a self-limited disease, and most patients return to a euthyroid state after a transient hyperthyroid state.

Option A (HLA-DR3 allele) is incorrect. This HLA antigenic type is associated with development of Graves disease.

Option B (Idiosyncratic drug reaction) is incorrect. Drugs are not likely to produce inflammation of the thyroid gland.

Option C (Iodine deficiency) is incorrect. Iodine deficiency would lead to a goiter with enlarged colloid-filled follicles lined by inactive flattened epithelium.

Page 56: usmle questionnarie

Option D (Prior radiation exposure) is incorrect. Radiation to the region of the neck encompassing the thyroid gland is a risk for thyroid carcinoma.

Option E (RET gene mutation) is incorrect. RET gene mutations are associated with multiple endocrine neoplasia (MEN) IIa or IIb. Medullary carcinoma of the thyroid gland may occur in this setting.

Pathogenesis. Subacute thyroiditis is believed to be caused by a viral infection or a postviral inflammatory process. The majority of patients have a history of an upper respiratory infection just before the onset of thyroiditis. The disease has a seasonal incidence, with occurrences peaking in the summer, and clusters of cases have been reported in association with coxsackievirus, mumps, measles, adenovirus, and other viral illnesses. Although the pathogenesis of the disease is unclear, one model suggests that it results from a viral infection that provides an antigen, either viral or a thyroid antigen that is released secondary to virus-induced host tissue damage. This antigen stimulates cytotoxic T lymphocytes, which then damage thyroid follicular cells. In contrast to autoimmune thyroid disease, the immune response is virus-initiated and not self-perpetuating, so the process is limited.

From Robbins & Cotran Pathologic Basis of Disease 7E by Kumar et al

A 31-year-old man has had a feeling of heaviness in his scrotum for over 6 months. The

examining physician notes an enlarged right testis. An ultrasound reveals a solid 5-cm mass in

the body of the right testis. Laboratory studies show a serum α-fetoprotein (AFP) of 81 ng/mL

and human chorionic gonadotropin (HCG) of 15 IU/L. A right orchiectomy is performed, and on

gross examination the testicular mass is soft and reddish brown. Microscopic examination shows

cords and sheets of primitive cells with large nuclei. Which of the following is the most likely

diagnosis?

A. Choriocarcinoma

B. Embryonal carcinoma

C. Leydig cell tumor

D. Squamous cell carcinoma

E. Teratoma

F. Yolk sac tumo

QID: 95485

Option B (Embryonal carcinoma) is correct. The embryonal carcinoma is likely to have an elevated AFP. Many malignant testicular neoplasms produce some detectable HCG, but this does not mean that choriocarcinoma is present.

Page 57: usmle questionnarie

Option A (Choriocarcinoma) is incorrect. Choriocarcinomas have primitive-appearing syncytiotrophoblastic cells in sheets with abundant hemorrhage. The serum HCG level is typically quite high.

Option C (Leydig cell tumor) is incorrect. Leydig cell tumors are not common. Most are small brown masses.

Option D (Squamous cell carcinoma) is incorrect. Squamous cell carcinomas do not arise in the testis.

Option E (Teratoma) is incorrect. A pure teratoma in the testis is very uncommon. Usually a malignant component such as embryonal carcinoma is present as well as part of a mixed germ cell tumor.

Option F (Yolk sac tumo) is incorrect. Yolk sac tumor is typically present in the pediatric age group.

Morphology. Grossly, the tumor is smaller than seminoma and usually does not replace the entire testis. On cut surfaces, the mass is often variegated, poorly demarcated at the margins, and punctuated by foci of hemorrhage or necrosis. Extension through the tunica albuginea into the epididymis or cord is not infrequent. Histologically, the cells grow in alveolar or tubular patterns, sometimes with papillary convolutions. Embryonal carcinomas lack the well-formed glands with basally situated nuclei and apical cytoplasm seen in teratomas. More undifferentiated lesions may present sheets of cells. The neoplastic cells have an epithelial appearance and are large and anaplastic, with hyperchromatic nuclei having prominent nucleoli. In contrast to seminoma, the cell borders are usually indistinct, and there is considerable variation in cell and nuclear size and shape. Mitotic figures and tumor giant cells are frequent. Within this background, syncytial cells containing HCG, cells containing AFP, or both may be detected by immunoperoxidase techniques. Most authorities allow for focal AFP positivity within an embryonal carcinoma without classifying the tumor as a mixed tumor. However, some purists designate any AFP positivity in an embryonal carcinoma, even if unaccompanied by yolk sac differentiation on the H&E stained section, as focal yolk sac tumor in a mixed tumor.

From Robbins & Cotran Pathologic Basis of Disease 7E by Kumar et al

A 10-year-old girl has developed increasing weakness over the past year. She has no muscle

pain or atrophy. She has epilepsy. Other family members are affected as shown by the pedigree

in the image. On physical examination she exhibits ataxia. A gastrocnemius biopsy is performed

and on microscopic examination shows increased numbers of ragged red fibers. Later in life, she

Page 58: usmle questionnarie

suffers a “stroke” at age 20. Which of the following conditions is she most likely to have?

A. Becker muscular dystrophy

B. Channelopathy

C. Lambert-Eaton syndrome

D. Mitochondrial myopathy

E. Myotonic dystrophy

QID: 95609

Option D (Mitochondrial myopathy) is correct. This is a mitochondrial myopathy known as MELAS (mitochondrial encephalopathy, lactic acidosis, and strokelike episodes). Mitochondrial genes are inherited from mother to children. Affected fathers cannot pass the trait.

Option A (Becker muscular dystrophy) is incorrect. Becker muscular dystrophy is a milder form of Duchenne muscular dystrophy, with an X-linked recessive inheritance pattern.

Option B (Channelopathy) is incorrect. Channelopathies are characterized by myotonia and periodic paralysis with an onset usually in childhood.

Option C (Lambert-Eaton syndrome) is incorrect.Lambert-Eaton syndrome is a rare form of paraneoplastic syndrome resembling myasthenia gravis. In most cases, a small cell anaplastic carcinoma of the lung is present.

Option E (Myotonic dystrophy) is incorrect. Myotonic dystrophy has an autosomal dominant pattern of inheritance.

Clinical Course and Genetics. The relationship between clinical course in the mitochondrial disorders and the genetic alterations is not entirely clear; however, three general categories have been defined. One set of mutations consists of point mutations in

Page 59: usmle questionnarie

mtDNA. These disorders tend to show a maternal pattern of inheritance, and some examples include myoclonic epilepsy with ragged red fibers (MERRF), Leber hereditary optic neuropathy (LHON), and mitochondrial encephalomyopathy with lactic acidosis and strokelike episodes (MELAS). A second set of mutations involves genes encoded by nuclear DNA and shows autosomal-dominant or autosomal-recessive inheritance. Some cases of subacute necrotizing encephalopathy (Leigh syndrome), exertional myoglobinuria, and infantile X-linked cardioskeletal myopathy (Barth syndrome) are due to mutations in nuclear DNA. The final subset of mitochondrial myopathies is caused by deletions or duplications of mtDNA. Examples include chronic progressive external ophthalmoplegia, characterized by a myopathy with prominent weakness of external ocular movements. Kearns-Sayre syndrome, another myopathy in this group, is also characterized by ophthalmoplegia but, in addition, includes pigmentary degeneration of the retina and complete heart block.

From Robbins & Cotran Pathologic Basis of Disease 7E by Kumar et al

A 45-year-old woman feels a “lump” in her left breast. Her physician palpates a 2-cm irregular

area in the upper outer quadrant. A biopsy is performed and microscopic examination shows no

evidence for carcinoma. Which of the following microscopic findings in this biopsy is most likely

to suggest an increased risk for subsequent development of breast carcinoma?

A. Fat necrosis

B. Galactocele

C. Lobular epithelial hyperplasia

D. Multiple cysts

E. Sclerosing adenosis

QID: 95524

Option C (Lobular epithelial hyperplasia) is correct. Particularly when atypical microscopic features are present, lobular or ductal hyperplasia is the component of fibrocystic changes that is associated with subsequent increased risk for development of carcinoma.

Option A (Fat necrosis) is incorrect. This lesion may sometimes be difficult to distinguish grossly from carcinoma, but it does not predispose to carcinoma.

Option B (Galactocele) is incorrect. This is a complication of obstruction to lactiferous sinuses during lactation. It may rarely occur with oral contraceptive use. It is not premalignant.

Option D (Multiple cysts) is incorrect.Simple cysts with fibrocystic disease are not associated with an increased risk for carcinoma.

Page 60: usmle questionnarie

Option E (Sclerosing adenosis) is incorrect. This component of fibrocystic disease does not imply an increased risk for breast cancer.

Epithelial Hyperplasia. In the normal breast, only myoepithelial cells and a single layer of luminal cells are present above the basement membrane. Epithelial hyperplasia is defined by the presence of more than two cell layers. Hyperplasia is moderate to florid when there are more than four cell layers. The proliferating epithelium, often including both luminal and myoepithelial cells, fills and distends the ducts and lobules. Irregular lumens (fenestrations) can usually be discerned at the periphery of the cellular masses.

From Robbins & Cotran Pathologic Basis of Disease 7E by Kumar et al

A 25-year-old woman has the sudden onset of fever, malaise, and nausea. On physical

examination she is afebrile. Her blood pressure is 160/90 mmHg. She has presacral edema. A

routine urinalysis shows 1&plus; protein, no ketones, no glucose, and 4&plus; blood, with RBC

casts seen on urine microscopic examination. A renal biopsy is performed; the light microscopic

findings are shown in the image. By immunofluorescence there is granular deposition of IgG and

C3 in glomerular capillary basement membranes. By electron microscopy there are electron-

dense subepithelial “humps.” Infection with which of the following organisms most likely preceded

development of her renal disease?

Courtesy of Dr. John A. Blackmon, Florida State University College of Medicine.

A. Candida albicans

B. Group A streptococus

C. Hepatitis B

D. Mycobacterium tuberclosis

E. Schistosoma hematobium

Page 61: usmle questionnarie

QID: 95456

Option B (Group A streptococus) is correct. Postinfectious glomerulonephritis (GN) is one of many causes for a nephritic syndrome, characterized by urinalysis showing hematuria and RBC casts. On light microscopy the glomeruli are hypercellular, with neutrophils present. Most cases occur in children, though adults can also be affected. Most children recover completely, though 1 in 5 adults may go on to develop chronic renal failure. Some cases of postinfectious GN may follow a streptococcal pharyngitis (poststreptococcal GN).

Option A (Candida albicans) is incorrect. Candida urinary tract infections can occur when patients are immunocompromised or very ill.

Option C (Hepatitis B) is incorrect. Hepatitis B infection is more likely to precede some cases of membranous GN.

Option D (Mycobacterium tuberculosis) is incorrect.Tuberculosis may be a rare cause for a “sterile pyuria” in which routine cultures fail to grow bacterial organisms, but inflammation is present.

Option E (Schistosoma hematobium) is incorrect. Schistosomiasis may lead to bladder inflammation and carcinoma.

Acute Proliferative (Poststreptococcal, Postinfectious) Glomerulonephritis

As the name implies, this cluster of diseases is characterized histologically by diffuse proliferation of glomerular cells, associated with influx of leukocytes. These lesions are typically caused by immune complexes. The inciting antigen may be exogenous or endogenous. The prototypic exogenous antigen-induced disease pattern is postinfectious glomerulonephritis, whereas that produced by an endogenous antigen is the nephritis of systemic lupus erythematosus. The most common infections are streptococcal, but the disorder has also been associated with other infections.

From Robbins & Cotran Pathologic Basis of Disease 7E by Kumar et al

A 50-year-old man has a past medical history including paresthesias, difficulty moving one or

more extremities, loss of sensation, and ataxia over the past 22 years. These problems have

waxed and waned. He now has developed paraplegia and incontinence. His brain T2-weighted

MR image is shown. Which of the following conditions involving his CNS is most likely to explain

Page 62: usmle questionnarie

the course of his disease?

A. Meningioma

B. Multiple sclerosis

C. Progressive multifocal leukoencephalopathy (PML)

D. Traumatic injury

E. Wernicke disease

QID: 95622

Option B (Multiple sclerosis) is correct. The history is most consistent with multiple sclerosis, a demyelinating disease that has exacerbations and remissions, but in most affected persons is eventually progressive. Note the bright plaques of demyelination involving the white matter of centrum semiovale.

Option A (Meningioma) is incorrect. A meningioma is slow growing but has progressive enlargement. The mass effect does not produce the range of findings listed.

Option C (Progressive multifocal leukoencephalopathy (PML)) is incorrect. PML usually occurs in patients with a history of immunosuppression (mostly AIDS). They do not survive 22 years.

Page 63: usmle questionnarie

Option D (Traumatic injury) is incorrect.A traumatic injury is not consistent with the range and variability of his findings. His neurologic problems are not service connected.

Option E (Wernicke disease) is incorrect. The findings do not point to a complication of chronic alcoholism. Wernicke disease is a consequence of thiamine deficiency. There are hemorrhages into the mammillary bodies and periaqueductal gray matter.

Morphology. Since MS is a white matter disease and gray matter covers much of the surface of the hemispheres, macroscopic examination of the outer aspect of the cerebral gyri is unremarkable. On the other hand, evidence of the disease may be found on the surface of the brainstem (e.g., basis pontis) or along the spinal cord, where myelinated fiber tracts course superficially; here lesions appear as multiple, well-circumscribed, somewhat depressed, glassy, gray-tan, irregularly shaped plaques, both on external examination and on section. In the fresh state, these have firmer consistency than the surrounding white matter (sclerosis). Plaques can be found throughout the white matter of the neuraxis; they may also extend into the gray matter structures, as these have myelinated fibers running through them, although their recognition within these regions is more difficult. The size of lesions varies considerably, from small foci that are recognizable only at microscopic examination to confluent plaques that involve large portions of the centrum semiovale. Plaques commonly occur beside the lateral ventricles and may be demonstrated to follow the course of paraventricular veins when the surface of the ventricle is inspected en face. They are also frequent in the optic nerves and chiasm, brain stem ascending and descending fiber tracts, cerebellum, and spinal cord.

From Robbins & Cotran Pathologic Basis of Disease 7E by Kumar et al

A 60-year-old woman develops severe chest pain over the past 6 hours. On physical

examination she has tachycardia with hypotension. Laboratory studies show an elevated serum

creatine kinase MB fraction. A coronary angiogram is performed emergently and reveals greater

than 90% occlusion of the left circumflex artery. Which of the following cellular changes is most

indicative of an irreversible injury to her myocardial fibers?

A. Glycogen depletion

B. Increased cytoplasmic sodium

C. Nuclear karyorrhexis

D. Diminished intracellular pH

E. Cell membrane blebbing

Page 64: usmle questionnarie

QID: 95048

Option C (Nuclear karyorrhexis) is correct. Chromatin clumping is reversible, but dissolution with fragmentation (karyorrhexis) of the entire nucleus is not. Once the nucleus is no longer functional, the cell cannot survive long, particularly a highly metabolically active cell such as a muscle cell.

Option A (Glycogen depletion) is incorrect. Glycogen depletion can be reversed.

Option B (Increased cytoplasmic sodium) is incorrect. Changes in cellular ion concentrations can occur from a variety of cellular insults. A cell that is damaged may not have optimal sodium pump function, but this is not an irreversible injury.

Option D (Diminished intracellular pH) is incorrect.Acidosis suggests a reduction in oxidative metabolism but by itself is not irreversible.

Option E (Cell membrane blebbing) is incorrect. Such blebs are reversible and do not constitute an irreversible injury.

Morphology. Necrotic cells show increased eosinophilia attributable in part to loss of the normal basophilia imparted by the RNA in the cytoplasm and in part to the increased binding of eosin to denatured intracytoplasmic proteins. The necrotic cell may have a more glassy homogeneous appearance than that of normal cells, mainly as a result of the loss of glycogen particles. When enzymes have digested the cytoplasmic organelles, the cytoplasm becomes vacuolated and appears moth-eaten. Finally, calcification of the dead cells may occur. Dead cells may ultimately be replaced by large, whorled phospholipid masses called myelin figures. These phospholipid precipitates are then either phagocytosed by other cells or further degraded into fatty acids; calcification of such fatty acid residues results in the generation of calcium soaps. By electron microscopy, necrotic cells are characterized by overt discontinuities in plasma and organelle membranes, marked dilation of mitochondria with the appearance of large amorphous densities, intracytoplasmic myelin figures, amorphous osmiophilic debris, and aggregates of fluffy material probably representing denatured protein.

From Robbins & Cotran Pathologic Basis of Disease 7E by Kumar et al

Nuclear changes appear in the form of one of three patterns, all due to nonspecific breakdown of DNA. The basophilia of the chromatin may fade (karyolysis), a change that presumably reflects DNase activity. A second pattern (also seen in apoptotic cell death) is pyknosis, characterized by nuclear shrinkage and increased basophilia. Here the DNA apparently condenses into a solid, shrunken basophilic mass. In the third pattern, known as karyorrhexis, the pyknotic or partially pyknotic nucleus undergoes fragmentation. With the passage of time (a day or two), the nucleus in the necrotic cell totally disappears.

Page 65: usmle questionnarie

From Robbins & Cotran Pathologic Basis of Disease 7E by Kumar et al

Once the necrotic cells have undergone the early alterations described, the mass of necrotic cells may have several morphologic patterns. Although the terms are somewhat outmoded, they are routinely used and their meanings are understood by both pathologists and clinicians. When denaturation is the primary pattern, coagulative necrosis develops. In the instance of dominant enzyme digestion, the result is liquefactive necrosis; in special circumstances, caseous necrosis and fat necrosis may occur.

From Robbins & Cotran Pathologic Basis of Disease 7E by Kumar et al

A 41-year-old man has epigastric burning pain along with nausea following meals for the past 6

months. He has lost 2 kg (4.4 lb) during that time. He has this pain when eating, but not often in

between meals or at night. On physical examination there are no abnormal findings. Upper GI

endoscopy is performed and there are two sharply demarcated ulcerations, 0.8 cm and 1.6 cm in

diameter, located in the antrum. Which of the following drugs used by this man is most likely to

have caused his illness?

A. Acetaminophen E. Ferrous sulfate

B. Cimetidine F. Ibuprofen

C. Cefotaxime G. Isoniazid

D. Cocaine H. Omeprazole

QID: 95412

Option F (Ibuprofen) is correct. Ibuprofen is one of the nonsteroidal anti-inflammatory drugs (NSAIDs), which are the second most common cause for gastritis and gastric ulceration, after Helicobacter pylori infection. The pain of gastric peptic ulceration can be precipitated by food. Nausea and weight loss occur more commonly in gastric ulcer patients than in duodenal ulcer patients. The typical pain pattern of duodenal occurs 1 to 3 hours after eating and is often relieved by antacids or eating. Pain at night that wakens a patient from sleep occurs in 2/3 of duodenal ulcer patients.

Option A (Acetaminophen) is incorrect. Acetaminophen is unlikely to produce gastritis or peptic ulceration. It is best known as a cause for hepatic necrosis when ingested in large quantities.

Option B (Cimetidine) is incorrect. Cimetidine is an H2 blocker used for treatment of peptic ulcer disease, without a significant risk for complication of ulceration itself.

Option C (Cefotaxime) is incorrect. Cefotaxime is a broad-spectrum antibiotic that can alter the gastrointestinal flora, but is unlikely to have an effect in the stomach.

Page 66: usmle questionnarie

Option D (Cocaine) is incorrect. Cocaine is a powerful vasoconstrictor, but its effect on the GI tract is minimal.

Option E (Ferrous sulfate) is incorrect. Ferrous sulfate does not produce gastric ulceration.

Option G (Isoniazid) is incorrect. Isoniazid is not associated with gastric ulceration.

Option H (Omeprazole) is incorrect. Omeprazole is a proton pump inhibitor used to treat gastric ulceration.

Chronic use of NSAIDs suppresses mucosal prostaglandin synthesis; aspirin also is a direct irritant. Cigarette smoking impairs mucosal blood flow and healing. Alcohol has not been proved to directly cause peptic ulceration, but alcoholic cirrhosis is associated with an increased incidence of peptic ulcers. Corticosteroids in high dose and with repeated use promote ulcer formation. In some patients with duodenal ulcers, there is too-rapid gastric emptying, exposing the duodenal mucosa to an excessive acid load. Duodenal ulcer also is more frequent in patients with alcoholic cirrhosis, chronic obstructive pulmonary disease, chronic renal failure, and hyperparathyroidism. In the latter two conditions, hypercalcemia stimulates gastrin production and therefore acid secretion. Genetic influences appear to play no major role in peptic ulceration. Finally, there are compelling arguments that personality and psychological stress are important contributing factors, even though hard data on cause and effect are lacking. Indeed, we might develop ulcers by trying to fathom their cause(s).

From Robbins & Cotran Pathologic Basis of Disease 7E by Kumar et al

A 35-year-old woman is bothered by right eye pain and irritation for 1 day. On slit lamp

examination with fluorescein dye she has an irregular dendritic corneal ulcer. She is most likely

to have an infection with which of the following organisms?

A. Acanthamoeba

B. Candida albicans

C. Chlamydia trachomatis

D. Cytomegalovirus (CMV)

E. Herpes simplex virus

F. Hemophilus aegypticus

G. Pseudomonas aeruginosa

QID: 95641

Page 67: usmle questionnarie

Option E (Herpes simplex virus) is correct. These lesions can ulcerate acutely and lead to scarring with chronic blindness.

Option A (Acanthamoeba) is incorrect. This organism does not produce dendritic ulcers.

Option B (Candida albicans) is incorrect. The eye is an uncommon site for fungal infections.

Option C (Chlamydia trachomatis) is incorrect. C. trachomatis is the causative agent that can produce trachoma, which leads to corneal abrasion and scarring.

Option D (Cytomegalovirus (CMV)) is incorrect. CMV is most likely to produce a retinitis in immunocompromised patients.

Option F (Hemophilus aegypticus) is incorrect. H. aegypticus can produce conjunctival inflammation, but not dendritic corneal ulcers.

Option G (Pseudomonas aeruginosa) is incorrect. P. aeruginosa infection does not typically produce a dendritic ulcer.

KERATITIS AND ULCERS

Various pathogens-bacterial, fungal, viral (especially herpes simplex and herpes zoster), and protozoal (Acanthamoeba) may cause corneal ulceration. In all forms of keratitis, dissolution of the corneal stroma may be accelerated by activation of collagenases within corneal epithelium and stromal fibroblasts (also known as keratocytes). Exudate and cells leaking from iris and ciliary body vessels into the anterior chamber may be visible by slit-lamp examination and may accumulate in sufficient quantity to become visible even by a penlight examination (hypopyon). Although the corneal ulcer may be infectious, the hypopyon seldom contains organisms and is an example par excellence of the vascular response to acute inflammation. The specific forms of keratitis may have certain distinctive features. For example, chronic herpes simplex keratitis may be associated with a granulomatous reaction to Descemet's membrane.

From Robbins & Cotran Pathologic Basis of Disease 7E by Kumar et al

A 47-year-old woman has had episodes of right upper quadrant pain during the past 2 weeks.

Her stools have become pale in color over the past 3 days. Laboratory studies show her serum

total bilirubin is 9.7 mg/dL. A cholangiogram shows that a gallstone has passed into the common

bile duct. Which of the following cellular alterations is most likely to be visualized on her skin

surfaces?

A. Calcification

B. Hemosiderosis

C. Icterus

D. Lipofuscin deposition

E. Steatosis

Page 68: usmle questionnarie

QID: 95436

Option C (Icterus) is correct. She probably has a “jaundiced” appearance to her sclerae and skin due to the increased amount of bilirubin. The bile pigments impart a yellow color to the tissues. She has biliary tract obstruction from cholelithiasis and choledocholithiasis.

Option A (Calcification) is incorrect. Dystrophic calcification can occur in areas of tissue damage, as in granulomatous diseases. The liver is not a typical spot for metastatic calcification.

Option B (Hemosiderosis) is incorrect. Excessive iron can be accumulated through increased absorption, increased intake, or prolonged transfusion therapy.

Option D (Lipofuscin deposition) is incorrect.Steatosis occurs with direct injury to hepatocytes, not biliary tract obstruction

Option E (Steatosis) is incorrect. Fatty change (steatosis) is a process that occurs in the liver, but biliary tract obstruction does not typically cause it. Steatosis can occur in the setting of alcohol abuse. Nonalcoholic fatty liver is seen in patients with obesity and diabetes mellitus. An element of steatosis can often be present with chronic hepatitis.

CHOLEDOCHOLITHIASIS AND ASCENDING CHOLANGITIS

These conditions are considered together, since they frequently go hand in hand. Choledocholithiasis is defined as the presence of stones within the bile ducts of the biliary tree, as opposed to cholelithiasis (stones in the gallbladder). In Western nations, almost all biliary tract stones are derived from the gallbladder, although both cholesterol and pigmented stones can form de novo anywhere in the biliary tree. In Asia, there is a much higher incidence of primary stone formation within the biliary tree, usually pigmented as a result of the biliary tract infections noted earlier in the discussion of gallstones. Choledocholithiasis may be asymptomatic or may cause symptoms from (1) obstruction, (2) pancreatitis, (3) cholangitis, (4) hepatic abscess, (5) secondary biliary cirrhosis, and (6) acute calculous cholecystitis.

From Robbins & Cotran Pathologic Basis of Disease 7E by Kumar et al

A 10-year-old girl has complained of joint pain for the past 2 years. The pain is localized to her

knees and ankles; it occurs either with or without movement. The arthralgias are accompanied by

fever. On physical examination she has a temperature of 37.8° C (100° F). There is no deformity

of her knees. She has generalized lymphadenopathy. Which of the following is the most likely

diagnosis?

A. Gonococcal arthritis

B. Ankylosing spondylitis

Page 69: usmle questionnarie

C. Juvenile rheumatoid arthritis

D. Tuberculous arthritis

E. Gout

QID: 95596

Option C (Juvenile rheumatoid arthritis) is correct. Also known as Still disease, juvenile rheumatoid arthritis (JRA) can involve just a single joint or a few joints (oligoarticular) or more than five joints (polyarticular). About 5% of rheumatoid arthritis cases occur in persons less than 16 years of age. Large joints are more often involved than small joints with JRA.

Option A (Gonococcal arthritis) is incorrect. Gonococcal arthritis is a suppurative arthritis, and it is seen most often in young persons as a complication of gonorrheal infection.

Option B (Ankylosing spondylitis) is incorrect. Ankylosing spondylitis typically involves the spine. It is slowly progressive, beginning in the third decade, and does not have a febrile component or lymphadenopathy.

Option D (Tuberculous arthritis) is incorrect.Tuberculous arthritis is a complication of adjacent osteomyelitis or dissemination. It is destructive and monoarticular, usually involving the spine.

Option E (Gout) is incorrect. Gouty arthritis usually involves the big toe of males with hyperuricemia who are past the third decade of life.

JRA differs from RA in adults in the following ways: (1) oligoarthritis is more common, (2) systemic onset is more frequent, (3) large joints are affected more often than small joints, (4) rheumatoid nodules and rheumatoid factor are usually absent, and (5) antinuclear antibody seropositivity is common. Pathogenetic factors, similar to those in RA, include genetic association with particular HLA haplotypes (DRB1); mycobacterial, bacterial, or viral infection; abnormal immunoregulation with the prevalence of activated CD4+ T cells within involved joints; and cytokine production. The morphologic changes in joint pathology are similar to those seen in adult RA.

From Robbins & Cotran Pathologic Basis of Disease 7E by Kumar et al

Commonly targeted joints are the knees, wrists, elbows, and ankles. They become warm and swollen and are often involved symmetrically. Pericarditis, myocarditis, pulmonary fibrosis, glomerulonephritis, uveitis, and growth retardation are potential extra-articular manifestations. A systemic onset may begin rather abruptly, associated with high spiking fevers, migratory and transient skin rash, hepatosplenomegaly, and serositis. Long-term studies of patients with jra show that one third to one half of patients have active disease when followed for at least 10 years.

Page 70: usmle questionnarie

From Robbins & Cotran Pathologic Basis of Disease 7E by Kumar et al

A 21-year-old woman has a routine Pap smear performed for a health screening examination.

The pathology report indicates that some cells are found cytologically to have larger, more

irregular nuclei. A follow-up cervical biopsy is shown in the image. No inflammatory cells are

present. Which of the following descriptive terms is best applied to these Pap smear and biopsy

findings?

Courtesy of Dr. David Cohen, Tel Aviv University

A. Anaplasia

B. Aplasia

C. Dysplasia

D. Hyperplasia

E. Metaplasia

QID: 95133

Option C (Dysplasia) is correct. The biopsy microscopically shows disordered maturation of the squamous epithelium, with hyperchromatic and pleomorphic nuclei extending from the basement membrane to half the thickness of the epithelial surface. Disordered growth of an epithelium is seen in dysplasia, which can be a precursor to neoplasia. Cervical squamous dysplasias most likely occur as a consequence of human papillomavirus (HPV) infection.

Page 71: usmle questionnarie

Option A (Anaplasia) is incorrect. Anaplasia refers to such an irregular appearance of cells in a neoplasm that it cannot be determined what kind of cell is present.

Option B (Aplasia) is incorrect. Aplasia refers to absence of growth, that is, lack of development, which is typically a congenital problem.

Option D (Hyperplasia) is incorrect.Metaplasia refers to exchange of one type of epithelium to another not typically found here. The endocervix, which normally has columnar epithelium, may undergo squamous metaplasia with persistent cervicitis.

Option E (Metaplasia) is incorrect. Hyperplasia refers to an increased number of cells.

Before we leave the subject of differentiation and anaplasia, we should discuss dysplasia, a term that literally means disordered growth. Dysplasia is encountered principally in epithelia, and it is characterized by a constellation of changes that include a loss in the uniformity of the individual cells as well as a loss in their architectural orientation. Dysplastic cells also exhibit considerable pleomorphism and often contain hyperchromatic nuclei that are abnormally large for the size of the cell. Mitotic figures are more abundant than usual, although almost invariably they conform to normal patterns. Frequently the mitoses appear in abnormal locations within the epithelium. Thus, in dysplastic stratified squamous epithelium, mitoses are not confined to the basal layers and may appear at all levels and even in surface cells. The architecture of the tissue may be disorderly. For example, the usual progressive maturation of tall cells in the basal layer to flattened squames on the surface may be lost and replaced by a scrambling of dark basal-appearing cells throughout the epithelium. When dysplastic changes are marked and involve the entire thickness of the epithelium, but the lesion remains confined to the normal tissue, it is considered a preinvasive neoplasm and is referred to as carcinoma in situ. Once the tumor cells move beyond the normal confines, the tumor is said to be invasive. Dysplastic changes are often found adjacent to foci of invasive carcinoma, and in some situations, such as in long-term cigarette smokers and Barrett esophagus, severe epithelial dysplasia frequently antedates the appearance of cancer. However, dysplasia does not necessarily progress to cancer. Mild to moderate changes that do not involve the entire thickness of epithelium may be reversible, and with removal of the inciting causes, the epithelium may revert to normal.

From Robbins & Cotran Pathologic Basis of Disease 7E by Kumar et al

An autopsy study reveals that evidence for atheroma formation can begin even in children. The

gross appearances of the aortas are recorded and compared with microscopic findings of

atheroma formation. Which of the following lesions shown in the image is most likely to be the

Page 72: usmle questionnarie

first visible gross evidence for the formation of an atheroma?

A. Calcification

B. Exudate

C. Fatty streak

D. Hemorrhage

E. Thrombus

F. Ulceration

QID: 95293

Option C (Fatty streak) is correct. This is the first sign of more to come. The fatty streak is benign and reversible, but it may be the precursor to more severe plaques if risk factors are present and continue.

Option A (Calcification) is incorrect. Calcification is a complication of an atheroma that is well established.

Option B (Exudate) is incorrect. An exudate is typically formed with a preponderance of acute inflammatory cells (neutrophils). Inflammatory cells, including lymphocytes and macrophages, play a role in atherogenesis, but their numbers are relatively small.

Option D (Hemorrhage) is incorrect. Hemorrhage is a complication of an atheroma that has already developed.

Option E (Thrombus) is incorrect. Thrombosis is a complication of an atheroma that is already present.

Option F (Ulceration) is incorrect. Ulceration of an atheromatous plaque is an advanced

Page 73: usmle questionnarie

lesion, not an early lesion. Ulceration or fissuring of the plaque increases the likelihood for overlying thrombosis.

Fatty streaks are the earliest lesion of atherosclerosis. They are composed of lipid-filled foam cells. They are not significantly raised and thus do not cause any disturbance in blood flow. Fatty streaks begin as multiple yellow, flat spots less than 1 mm in diameter that coalesce into elongated streaks, 1 cm long or longer. They contain T lymphocytes and extracellular lipid in smaller amounts than in plaques.

From Robbins & Cotran Pathologic Basis of Disease 7E by Kumar et al

A 38-year-old man has had scaling patches on his elbows, knees, and scalp for the past 6

months. On physical examination the lesions have the appearance shown. In addition he has

yellow-brown discoloration with thickening and pitting of his fingernails. A punch biopsy of one

lesion is performed and on microscopic examination shows thinning of the stratum granulosum,

downward projection of the rete pegs, small neutrophilic aggregates in the superficial epidermis,

and marked overlying parakeratosis. Therapy with methotrexate and UVA light reduces the size

of the lesions. Which of the following is the most likely diagnosis?

From Klatt E: Robbins and Cotran Atlas of Pathology, 2nd ed. Philadelphia, Elsevier, 2010.

A. Bullous pemphigoid

B. Dermatitis herpetiformis

C. Erythema multiforme (EM)

D. Lichen planus

E. Pemphigus vulgaris

F. Psoriasis

Page 74: usmle questionnarie

QID: 95572

Option F (Psoriasis) is correct. Psoriasis affects 1% to 2% of persons in the United States. Genetic and autoimmune mechanisms have been implicated in its pathogenesis. There may be accompanying arthritis and enteropathy, among other manifestations. Grossly, the scaling lesions have a salmon pink to silvery gray appearance.

Option A (Bullous pemphigoid) is incorrect. Bullous pemphigoid is a rare condition producing flaccid bullae in older persons on inner thighs and flexor surfaces. It is a type II hypersensitivity reaction with immunoglobulin and complement deposited along the epidermal basement membrane and directed at hemidesmosomes.

Option B (Dermatitis herpetiformis) is incorrect. Dermatitis herpetiformis has urticaria of grouped vesicles on extensor surfaces in young adults, often associated with celiac disease. IgA antibodies are implicated in the pathogenesis.

Option C (Erythema multiforme (EM)) is incorrect. EM is thought to be a hypersensitivity response to drugs or infection. The lesions can be macules, papules, vesicles, or bullae. There is often a target-like appearance to the erythema. EM is usually self-limited.

Option D (Lichen planus) is incorrect. Lichen planus is a self-limited disease that usually disappears in a year or two. There are pruritic, flattened, purplish papules on the skin, and white to reticulated areas on mucous membranes. Microscopically, there is a dense, bandlike upper dermal infiltrate of lymphocytes.

Option E (Pemphigus vulgaris) is incorrect. Blisters resulting from a type II hypersensitivity reaction with IgG antibodies directed at desmoglein that glues keratinocytes together characterize pemphigus vulgaris.

Clinically, psoriasis most frequently affects the skin of the elbows, knees, scalp, lumbosacral areas, intergluteal cleft, and glans penis. The most typical lesion is a well-demarcated, pink to salmon-colored plaque covered by loosely adherent scales that are characteristically silver-white in color. Variations exist, with some lesions occurring in annular, linear, gyrate, or serpiginous configurations. Psoriasis can be one cause of total body erythema and scaling known as erythroderma. Nail changes occur in 30% of cases of psoriasis and consist of yellow-brown discoloration (often likened to an oil slick), with pitting, dimpling, separation of the nail plate from the underlying bed (onycholysis), thickening, and crumbling. In the rare variant called pustular psoriasis, multiple small pustules form on erythematous plaques. This type of psoriasis is either benign and localized (hands and feet) or generalized and life-threatening, with associated fever, leukocytosis, arthralgia, diffuse cutaneous and mucosal pustules, secondary infection, and electrolyte disturbances.

From Robbins & Cotran Pathologic Basis of Disease 7E by Kumar et al